New Research

The Physics of Why Hot Water Sometimes Freezes Faster Than Cold Water

For decades, physicists have debated whether the phenomenon exists and how to study it

Theresa Machemer

Correspondent

Apparatus

The story goes that in 1963, Tanzanian high school student Erasto Mpemba was making ice cream with his class when he impatiently put his sugar and milk concoction into the ice cream churner when it was still hot, instead of letting it cool first. To his surprise, the confection cooled faster than his classmates’ had.

With the help of a physics professor, Mpemba performed additional experiments by putting two glasses of water, one just-boiled and one warm, in a freezer, and seeing which one reached the freezing finish line first. Often, the water with a higher starting temperature was the first to freeze. Their observations set off a decades-long discussion over the existence and details of the counterintuitive phenomenon, now called the Mpemba effect.

Now, new research published on August 5 in the journal Nature not only shows that the Mpemba effect does exist, but also sheds light on how it occurs, Emily Conover reports for Science News .

Rather than experiment on freezing water, which is surprisingly complicated to study, physicists Avinash Kumar and John Bechhofer of Simon Fraser University focused their sights—and lasers—on microscopic glass beads. They measured how the glass beads moved under very specific conditions in water and saw that in some circumstances, beads that started off very hot cooled faster than those that didn’t.

“It’s one of these very simple setups, and it already is rich enough to show this effect.” University of Virginia theoretical physicist Marija Vucelja tells Science News . The experiment also suggests that the effect might show up in materials other than water and glass beads. Vucelja says, “I would imagine that this effect appears quite generically in nature elsewhere, just we haven’t paid attention to it.”

If the freezing point is the finish line, then the initial temperature is like the starting point. So it would make sense if a lower initial temperature, with less distance to the finish line, is always the first to reach it. With the Mpemba effect, sometimes the hotter water reaches the finish line first.

But it gets more complicated. For one thing, water usually has other stuff, like minerals, mixed in. And physicists have disagreed over the what exactly the finish line is: is it when the water in a container reaches the freezing temperature, begins to solidify, or completely solidifies? These details make the phenomenon hard to study directly, Anna Demming writes for Physics World .

The new experiment does away with the details that make the Mpemba effect so murky. In each test, they dropped one microscopic glass bead into a small well of water. There, they used a laser to exert controlled forces on the bead, and they measured the bead’s temperature, per Science News . They repeated the test over 1,000 times, dropping the beads in different wells and starting at different temperatures.

Under certain forces from the laser, the hottest beads cooled faster than the lower temperature beads. The research suggests that the longer path from a higher temperature to the freezing point might create shortcuts so that the hot bead’s temperature can reach the finish line before the cooler bead.

Bechhoefer describes the experimental system as an “abstract” and “almost geometrical” way to picture the Mpemba effect to Physics World . But using the system, he and Kumar identified the optimal “initial temperatures” for a Mpemba cooling effect.

“It sort of suggested that all the peculiarities of water and ice – all the things that made the original effect so hard to study – might be in a way peripheral,” Bechhoefer tells Physics World .

Get the latest stories in your inbox every weekday.

Theresa Machemer | READ MORE

Theresa Machemer is a freelance writer based in Washington DC. Her work has also appeared in National Geographic and SciShow. Website: tkmach.com

 alt=

  • Undergraduate
  • Postdoctoral Programs
  • Future Engineers
  • Professional Education
  • Open Access
  • Global Experiences
  • Student Activities
  • Leadership Development
  • Graduate Student Fellowships
  • Aeronautics and Astronautics
  • Biological Engineering
  • Chemical Engineering
  • Civil and Environmental Engineering
  • Electrical Engineering and Computer Science
  • Institute for Medical Engineering and Science
  • Materials Science and Engineering
  • Mechanical Engineering
  • Nuclear Science and Engineering
  • Industry Collaborations
  • Engineering in Action
  • In The News
  • Video Features
  • Newsletter: The Infinite
  • Ask an Engineer
  • Facts and Figures
  • Diversity, Equity & Inclusion
  • Staff Spotlights
  • Commencement 2024

Ask An Engineer

Does hot water freeze faster than cold water?

Related Questions

  • Why doesn’t a plain, white piece of paper reflect light, but a mirror does?
  • Does a golf ball really change its shape when struck by the club?
  • How does glass change over time?
  • How can a snail crawl upside-down on the underside of the surface of a pond?
  • Why is mercury liquid at room temperature?
  • How does a match burn in a spacecraft?
  • Are there materials that can absorb heat without becoming hot?
  • What are the basic forces behind tape and glue?
  • What’s the difference between premium-grade and regular gasoline?
  • What makes wood rot so slowly?

Does hot water freeze faster than cold water?

It’s an age-old question with a simple answer: no.

Since the time of Aristotle, researchers and amateur scientists alike have batted about the counterintuitive theory that hot water freezes faster than cold. The notion even has a name: the Mpemba effect, named for a Tanzanian schoolboy who in 1963 noticed that the ice cream he and his classmates made from warm milk froze quicker than that made from cool milk.

“No matter what the initial temperature of water is, it must be brought to the freezing point before it will change state and become ice,” says Prakash Govindan, a postdoctoral associate in MIT’s mechanical engineering department. It will actually take more time and/or energy to freeze hot water because it must be brought down further in temperature until it reaches the freezing point, about 0 ° C.

Govindan suggests conducting a simple experiment to demonstrate that hot and cold water will behave as logic predicts. “Fill two identical containers with hot and cold tap water from the kitchen sink and see which freezes first,” he says. Interestingly, he points out, the rates of change in this experiment will not be the same. “When you set them in the freezer, the freezer will work harder to bring the temperature of the hot water down, so initially the rate of heat transfer will be faster in the hot water.” However, the other container will be cooling at the same time (if not at quite the same rate).

When the temperature of the water in each container reaches just about 0 ° C it will undergo the same changes as it moves from a liquid to a solid, and it will take the same amount of time to begin forming tiny ice crystals. At that point, each mixture of liquid and ice will be at a uniform temperature, and as more heat is taken from the mixtures, the thermodynamic principle of latent heat kicks in: The water continues to convert to a solid state, but no longer changes in temperature. “As long as you have a mixture of liquid water and solid ice, the temperature will remain at 0 until all the water is frozen,” says Govindan.

It’s never been convincingly proven than hot water and cold water behave differently from each other at any step of the freezing process, despite the ongoing fascination with the Mpemba effect. In early 2013, Europe’s Royal Society of Chemistry even held a competition for the best explanation of the theory. The winner speculated that hot water indeed freezes more quickly if the cold water is first supercooled. But logic triumphs when it comes the plain ordinary water that comes from the household faucet. Most likely to impact the freezing point of water is the presence of impurities such as salt, dissolved solids and gases — and the ingredients of homemade ice cream. 

Thanks to Khubaib Mukhtar of Pakistan for this question.

April 30, 2013

 alt=

October 21, 1998

Is It True that Hot Water Freezes Faster than Cold Water or that Cold Water Boils Faster than Hot Water?

It seems hard tobelieve, but some people swear that it is so

does hot water or cold water freeze faster experiment

A woman tosses hot water into the freezing cold air.

Ismail Kaplan Getty Images

This seemingly simple question continues to generate considerable controversy. Takamasa Takahashi, a physicist at St. Norbert College in De Pere, Wis., attempts a definitive answer:

"Cold water does not boil faster than hot water. The rate of heating of a liquid depends on the magnitude of the temperature difference between the liquid and its surroundings (the flame on the stove, for instance). As a result, cold water will be absorbing heat faster while it is still cold; once it gets up to the temperature of hot water, the heating rate slows down and from there it takes just as long to bring it to a boil as the water that was hot to begin with. Because it takes cold water some time to reach the temperature of hot water, cold water clearly takes longer to boil than hot water does. There may be some psychological effect at play; cold water starts boiling sooner than one might expect because of the aforementioned greater heat absorption rate when water is colder.

"To the first part of the question--'Does hot water freeze faster than cold water?'--the answer is 'Not usually, but possibly under certain conditions.' It takes 540 calories to vaporize one gram of water, whereas it takes 100 calories to bring one gram of liquid water from 0 degrees Celsius to 100 degrees C. When water is hotter than 80 degrees C, the rate of cooling by rapid vaporization is very high because each evaporating gram draws at least 540 calories from the water left behind. This is a very large amount of heat compared with the one calorie per Celsius degree that is drawn from each gram of water that cools by regular thermal conduction.

On supporting science journalism

If you're enjoying this article, consider supporting our award-winning journalism by subscribing . By purchasing a subscription you are helping to ensure the future of impactful stories about the discoveries and ideas shaping our world today.

"It all depends on how fast the cooling occurs, and it turns out that hot water will not freeze before cold water but will freeze before lukewarm water. Water at 100 degrees C, for example, will freeze before water warmer than 60 degrees C but not before water cooler than 60 degrees C. This phenomenon is particularly evident when the surface area that cools by rapid evaporation is large compared with the amount of water involved, such as when you wash a car with hot water on a cold winter day. [For reference, look at Conceptual Physics, by Paul G. Hewitt (HarperCollins, 1993).]

"Another situation in which hot water may freeze faster is when a pan of cold water and a pan of hot water of equal mass are placed in a freezer compartment. There is the effect of evaporation mentioned above, and also the thermal contact with the freezer shelf will cool the bottom part of the body of water. If water is cold enough, close to four degrees C (the temperature at which water is densest), then near-freezing water at the bottom will rise to the top. Convection currents will continue until the entire body of water is 0 degrees C, at which point all the water finally freezes. If the water is initially hot, cooled water at the bottom is denser than the hot water at the top, so no convection will occur and the bottom part will start freezing while the top is still warm. This effect, combined with the evaporation effect, may make hot water freeze faster than cold water in some cases. In this case, of course, the freezer will have worked harder during the given amount of time, extracting more heat from hot water."

Robert Ehrlich of George Mason University, in Fairfax, Va., adds to some of the points made by Takahashi:

"There are two ways in which hot water could freeze faster than cold water. One way [described in Jearl Walker's book The Flying Circus of Physics (Wiley, 1975)] depends on the fact that hot water evaporates faster, so that if you started with equal masses of hot and cold water, there would soon be less of the hot water to freeze, and hence it would overtake the cold water and freeze first, because the lesser the mass, the shorter the freezing time. The other way it could happen (in the case of a flat-bottomed dish of water placed in a freezer) is if the hot water melts the ice under the bottom of the dish, leading to a better thermal contact when it refreezes." �

Still feeling skeptical? Fred W. Decker, a meteorologist at Oregon State University in Corvallis, encourages readers to settle the question for themselves:

"You can readily set up an experiment to learn which freezes earlier: water that is initially hot, or water that is initially cold. Use a given setting on an electric hot plate and clock the time between start and boiling for a given pot containing, say, one quart of water; first start with the water as cold as the tap will provide and then repeat it with the hottest water available from that tap. I'd wager the quart of water initially hot will come to a boil in much less time than the quart of water initially cold.

"The freezing experiment is harder to perform, because it ideally requires a walk-in cold storage chamber that is set to a temperature below freezing. Take into the chamber two quart-volume milk bottles filled with water, one from a hot tap and the other from a cold tap outside the chamber. Time them to freezing, and I would wager again that the initially colder water will freeze sooner than the initially hot water."

[We would add that, if you don't want to suffer in a walk-in freezer, you can conduct a reasonably good version of the above experiment in the freezer compartment of your refrigerator; just don't check the water too often-in which case it will never freeze-or too infrequently, in which case you may miss the moment when one container is frozen but not the other.]

Decker concludes that "much folklore results from trying to answer such a question under conditions that do not make 'all other things equal,' which the foregoing experiments do.

Why Does Hot Water Freeze Faster Than Cold Water?

does hot water or cold water freeze faster experiment

Lots of you in the Northern Hemisphere will be in the middle of yet another winter, and some might even be experiencing the sub-zero temperatures to do some cool experiments, such as creating giant frozen marbles for the front yard ,  snapping soap bubbles , or even tossing boiling water into air to create snow (although that last one requires caution, seriously).

If you remember the dreadfully cold winter of 2013, you might have seen videos like the one we've posted below circulating as 'proof' of the crazy low temperatures. But it turns out this demonstration is actually not that surprising - hot water is in fact known to freeze faster than cold water. There are records of this from as far back as Aristotle's time.

In modern times, this counterintuitive property has been named the Mpemba effect, after a Tanzanian secondary school student who re-discovered this phenomenon back in 1963.

Erasto Mpemba and other kids at his school often made ice cream using the school freezer - they would do this by boiling milk and mixing it with sugar, which then had to be cooled and placed in the freezer. One day, Mpemba rushed the process and stuck the milk in while still hot, and to his surprise, the ice cream formed quicker than for his classmate. Unsurprisingly, none of his teachers believed the 13-year-old.

Mpemba later teamed up with a physics professor who visited his high school, and in 1969 they published a paper , which has since been replicated many times - most often with a similar result. Although if you try this at home and fail, it's probably because the Mpemba effect is not a reliable phenomenon that happens every single time; in fact, there seem to be several factors at play.

So what are they? Well, it's one of those somewhat unsatisfying cases where just because we know something happens, doesn't mean we entirely understand why. But the process of scientific speculation is still interesting, so here are some versions.

The most commonly proposed hypothesis - and one that's probably somewhat responsible for the effect - is that hot water evaporates more quickly, losing mass and therefore needing to lose less heat in order to freeze. However, scientists have also demonstrated the Mpemba effect with closed containers where evaporation doesn't take place.

Another theoretical speculation is that water develops convection currents and temperature gradients as it cools - a rapidly cooling glass of hot water will have greater temperature differences throughout, and lose heat more quickly from the surface, whereas a uniformly cool glass of water has less of a temperature difference, and there's less convection to accelerate the process. But this idea has not been entirely verified either.

Other theories have been put forward, including supercooling, or the effect that dissolved gasses in the water would have on the freezing process. It's likely that several of these actually come into play.

In late 2013, a team of researchers from Singapore proposed in a paper on arXiv.org that the Mpemba paradox stems from the unique properties of chemical bonds in the water. A standard water molecule contains one oxygen atom and two hydrogen atoms joined to it by covalent bonds sharing electron pairs between the atoms.

But when you put several water molecules together, the hydrogen atoms will also form bonds with oxygen atoms in other molecules. These hydrogen bonds are what gives water some of its properties, such as having a relatively high boiling point, and becoming less dense when frozen.

According to the researchers, when water boils, the molecules spread out, lengthening the hydrogen bonds - but because volume is limited, meanwhile the covalent bonds within individual molecules get compressed, storing away energy. If water is frozen at this state, the bonds release energy as an uncoiled spring, cooling down much more quickly than if the covalent bonds were less compressed.

However, even though many headlines heralded this as the definitive explanation for the Mpemba effect, the idea is yet to be published in a peer-reviewed journal. According to the blog of Physics arXiv , the theory is convincing, but lacks predictive power:

"Xi and co need to use their theory to predict a new property of water that conventional thinking about water does not. For example, the shortened covalent bonds might give rise to some measurable property of the water that would not otherwise be present. The discovery and measurement of this property would be the coup de grâce  that their theory needs."

As Adam Mann wrote at Wired , this explanation is also not strongly supported in the scientific community. "When we talked to chemist Richard Zare of Stanford University, he was unconvinced of its merits and said he thinks the dominant force at play is evaporation."

Still, it's fun to (carefully) try this at home.

does hot water or cold water freeze faster experiment

Choose an Account to Log In

Roly

Notifications

Science project, does hot water freeze faster than cold water.

does hot water or cold water freeze faster experiment

Have you ever refilled the ice cube tray in your freezer after using the last ice cube in your cup of juice? You probably automatically poured cold water in the ice cube tray without asking the question, "Does hot water freeze faster than cold water?"

It makes sense to believe that cold water would turn to ice before hot water because the hot water would need to cool first before it could freeze; but how do you know if that idea is correct? Test this theory —untested idea—will tell you whether cold water actually freezes faster than hot water.

Does temperature affect how quickly water freezes?

  • 3 bowls of equal size and shape
  • Sticky labels
  • Measuring cup
  • Thermometer
  • Clear enough room in your freezer for the three bowls. You need to be able to put them in the freezer at exactly the same time, so you don't want to be moving your frozen food and drinks around later.
  • Think about what you know about ice. What temperature is water right before it freezes? You probably usually take baths in warm water. How quickly does the water turn cold when you're in the tub?
  • After considering different temperatures of water and ice, make a guess—called a hypothesis —answering the question: Does hot water freeze faster than cold water?
  • Write your hypothesis in your notebook, including whether you think the hot, warm, or cold water would freeze first and why .
  • Using your marker, write Hot on one of your sticky labels. Repeat with labels for Warm and Cold.
  • Place the sticky labels on each of the three bowls, using one per bowl. The labels will help you keep track of which bowl holds which temperature of water.
  • With your pencil, draw three columns in your notebook. Label the first column Hot, the second one Warm and the third Cold.
  • With the help of an adult, heat 1 cup of water to 100 degrees Fahrenheit. Pour it into the Hot bowl, being careful not to burn yourself.
  • Heat 1 cup of water to 70 degrees Fahrenheit, and pour it into the Warm bowl.
  • Fill the Cold bowl with water that's 40 degrees Fahrenheit.
  • Immediately place all three bowls in the freezer.
  • Record the starting temperatures in the correct columns of your notebook.
  • Open the freezer door every 10 minutes and take the temperature of the water in each bowl with a thermometer. Record the temperature in your notebook.
  • Repeat Step 13 until all three bowls have frozen over.
  • Compare the information in each of the three columns in your notebook. Was your hypothesis correct?

The bowls that contain the hot and warm water will freezer faster than the bowl that is filled with cold water.

Hot water freezing more quickly than cold water is known as the Mpemba effect . So, why does the Mpemba effect occur?

First, all water evaporates , which means that the liquid (water) "disappears" and becomes a vapor , or gas. Hot water evaporates at a much faster rate than cold water. This means that the bowl with hot water actually had less water than the bowl with cold water, which helped it freeze more quickly.

Second, convection (the transfer of heat within the water as it moves around) plays a part in helping hot water freeze more quickly than the bowl of cold water. The hot water has more convection currents than cold water, causing it to cool down much more quickly. That's why your bath water always seems to get cold much faster than you'd like!

Now that you know about freezing water at different temperatures, keep the science going by testing other liquids, such as milk or apple juice. Will warm milk freeze faster than cold milk? Or, switch up the project altogether! Does milk freeze faster than water at the same temperature? Science is all about guessing what will happen, then testing to see if you're right. You now know that hot water freezes faster than cold water, so brainstorm a new project that you're interested in. By constantly changing your experiments, you'll continue learning new things—and become a science whiz!

Related learning resources

Add to collection, create new collection, new collection, new collection>, sign up to start collecting.

Bookmark this to easily find it later. Then send your curated collection to your children, or put together your own custom lesson plan.

Every print subscription comes with full digital access

Science News

You really can freeze hot water faster than cold*.

*But only if you’re a clever physicist and you bend the rules

Share this:

By Laura Sanders

March 23, 2010 at 4:36 pm

Hot water really can freeze faster than cold water, a new study finds. Sometimes. Under extremely specific conditions. With carefully chosen samples of water.

New experiments provide support for a special case of the counterintuitive Mpemba effect, which holds that water at a higher temperature turns to ice faster than cooler water.

The Mpemba effect is named for a Tanzanian schoolboy, Erasto B. Mpemba, who noticed while making ice cream with his classmates that warm milk froze sooner than chilled milk. Mpemba and physicist Denis Osborne  published a report of the phenomenon in Physics Education in 1969. Mpemba joined a distinguished group of people who had also noticed the effect: Aristotle, Francis Bacon and René Descartes had all made the same claim. 

On the surface, the notion seems to defy reason. A container of hot water should take longer to turn into ice than a container of cold water, because the cold water has a head start in the race to zero degrees Celsius.

But under scientific scrutiny, the issue becomes murky. The new study doesn’t explain the phenomenon, but it does identify special conditions under which the Mpemba effect can be seen, if it truly exists.

“All in all, the work is a nice beginning, but not systematic enough to do more than confirm it can happen,” comments water expert David Auerbach, whose own experiments also suggest that the effect does occur.

Papers published over the last decade, including several by Auerbach, who performed his research while at the Max Planck Institute for Flow Research in Göttingen, Germany, have documented instances of  hot water freezing faster than cold, but not reproducibly, says study author James Brownridge of State University of New York at Binghamton. “No one has been able to get reproducible results on command.”

That’s what Brownridge has done. One of his experiments, presented online , repeatedly froze a sample of hot water faster than a similar sample of cool water.

Note the word similar . In order for the experiment to work, the cool water had to be distilled, and the hot water had to come from the tap.

In the experiment, about two teaspoons of each sample were held in a copper device that completely surrounded the water, preventing evaporation and setting reasonably even temperatures. Freezing was official when sensors picked up an electrical signal created by ice formation. Brownridge heated the tap water to about 100° C, while the distilled water was cooled to 25° C or lower. When both samples were put into the freezer, the hot water froze before the cold water. Brownridge then thawed the samples and repeated the experiment 27 times. Each time, the hot tap water froze first.

The experiment worked because the two types of water have different freezing points, Brownridge says. Differences in the shape, location and composition of impurities can all cause water’s freezing temperature — which in many cases is below zero degrees C — to vary widely. With a higher freezing point, the tap water had an edge that outweighed the distilled water’s lower temperature.

Because the experiment didn’t compare two identical samples of water, the mystery of the Mpemba effect is not really solved. “I’m not arrogant enough to say I’ve solved this,” Brownridge says. But he has set some guidelines about when the effect can be seen. Physical chemist Christoph Salzmann of the University of Durham in England says he’s not convinced the Mpemba effect really exists, because there are innumerable things that influence the timing of freezing, making it impossible to completely control.

Predicting how long it will take for a water sample to crystallize “is a bit like trying to predict when the next earthquake or crash of the stock market will happen,” he says. “I would not want to say that the Mpemba effect does not exist. But I have still not been convinced of its existence.”

More Stories from Science News on Physics

A swirl of two particles represents the tauonium atom in an illustration. The atom has emerged from a particle detector represented by a series of concentric cylinders, centered around a beam line where electrons and positrons enter from either side.

Scientists propose a hunt for never-before-seen ‘tauonium’ atoms 

Illustration showing three atoms, representing quantum memories, are connected by lines, representing entanglement, over a cityscape backdrop.

Two real-world tests of quantum memories bring a quantum internet closer to reality

An illustration shows water molecules arranged in hexagons and other shapes surrounded by ice

Here’s how ice may get so slippery 

A sensor chip with multiple small pixels is shown

The neutrino’s quantum fuzziness is beginning to come into focus

An illustration shows a doughnut shape filled with galaxies

The universe may have a complex geometry — like a doughnut

A lattice of gold-colored spheres, with each sphere connected by lines to six of its neighbors

Scientists developed a sheet of gold that’s just one atom thick

An illustration of atoms in an altermagnet shows a grid of alternating blue and purple shapes, rotated with respect to one another.

Newfound ‘altermagnets’ shatter the magnetic status quo 

A woman is pictured in front of three overlapping circles, representing the three stars of an alien star system, in an image from the Netflix show "3 Body Problem."

Separating science fact from fiction in Netflix’s ‘3 Body Problem’ 

Subscribers, enter your e-mail address for full access to the Science News archives and digital editions.

Not a subscriber? Become one now .

  • Comment Comments
  • Save Article Read Later Read Later

Controversy Continues Over Whether Hot Water Freezes Faster Than Cold

June 29, 2022

Whether hot or cold water freezes faster remains unknown.

Francis Chee / SPL / Science Source

Introduction

It sounds like one of the easiest experiments possible: Take two cups of water, one hot, one cold. Place both in a freezer and note which one freezes first. Common sense suggests that the colder water will. But luminaries including Aristotle, Rene Descartes and Sir Francis Bacon have all observed that hot water may actually cool more quickly. Likewise, plumbers report hot water pipes bursting in subzero weather while cold ones remain intact. Yet for more than half a century, physicists have been arguing about whether something like this really occurs.

The modern term for hot water freezing faster than cold water is the Mpemba effect, named after Erasto Mpemba, a Tanzanian teenager who, along with the physicist Denis Osborne, conducted the first systematic, scientific studies of it in the 1960s. While they were able to observe the effect, follow-up experiments have failed to consistently replicate that result. Precision experiments to investigate freezing can be influenced by many subtle details, and researchers often have trouble determining if they have accounted for all confounding variables.

Over the past few years, as the controversy continues about whether the Mpemba effect occurs in water, the phenomenon has been spotted in other substances — crystalline polymers , icelike solids called clathrate hydrates , and manganite minerals cooling in a magnetic field. These new directions are helping researchers peek into the complicated dynamics of systems that are out of thermodynamic equilibrium. A contingent of physicists modeling out-of-equilibrium systems has predicted the Mpemba effect should occur in a wide variety of materials (along with its inverse, in which a cold substance heats up faster than a warm one). Recent experiments appear to confirm these ideas.

Yet the most familiar substance of all, water, is proving to be the slipperiest.

“A glass of water stuck in a freezer seems simple,” said  John Bechhoefer , a physicist at Simon Fraser University in Canada whose recent experiments are the most solid observations of the Mpemba effect to date. “But it’s actually not so simple once you start thinking about it.”

‘That Cannot Happen’

“My name is Erasto B. Mpemba, and I am going to tell you about my discovery, which was due to misusing a refrigerator.” Thus begins a 1969 paper in the journal Physics Education in which Mpemba described an incident at Magamba Secondary School in Tanzania when he and his classmates were making ice cream.

A middle-aged man in a suit holding an ice cube in his palm

The late Erasto Mpemba initiated decades of research into whether hot water freezes faster than cold water.

PA Images / Alamy Stock Photo

Space was limited in the students’ refrigerator, and in the rush to nab the last available ice tray, Mpemba opted to skip waiting for his boiled-milk-and-sugar concoction to cool to room temperature like the other students had done. An hour and a half later, his mixture had frozen into ice cream, whereas those of his more patient classmates remained a thick liquid slurry. When Mpemba asked his physics teacher why this occurred, he was told, “You were confused. That cannot happen.”

Later, Osborne came to visit Mpemba’s high school physics class. He recalled the teenager raising his hand and asking, “If you take two beakers with equal volumes of water, one at 35°C and the other at 100°C, and put them into a refrigerator, the one that started at 100°C freezes first. Why?” Intrigued, Osborne invited Mpemba to the University College in Dar es Salaam, where they worked with a technician and found evidence for the effect that bears Mpemba’s name. Still, Osborne concluded that the tests were crude and more sophisticated experiments would be needed to figure out what might be going on.

Over the decades, scientists have offered a wide variety of theoretical explanations to explain the Mpemba effect. Water is a strange substance, less dense when solid than liquid, and with solid and liquid phases that can coexist at the same temperature. Some have suggested that heating water might destroy the loose network of weak polar hydrogen bonds between water molecules in a sample, increasing its disorder, which then lowers the amount of energy it takes to cool the sample. A more mundane explanation is that hot water evaporates faster than cold, decreasing its volume and thus the time it takes to freeze. Cold water also could contain more dissolved gases, which lower its freezing point. Or perhaps external factors come into play: A layer of frost in a freezer can act as an insulator, keeping heat from leaking out of a cold cup, whereas a hot cup will melt the frost and cool faster.

Those explanations all assume that the effect is real — that hot water really does freeze faster than cold. But not everyone is convinced.

In 2016, physicist Henry Burridge of Imperial College London and mathematician Paul Linden of the University of Cambridge did an experiment that showed how sensitive the effect is to the particulars of measurement. They speculated that hot water might form some ice crystals first but take longer to fully freeze. Both of these events are difficult to measure, so Burridge and Linden instead noted how long it took water to reach zero degrees Celsius. They found that the readings depended on where they placed the thermometer. If they compared the temperatures between hot and cold cups at the same height, the Mpemba effect didn’t appear. But if measurements were off by even a centimeter, they could produce false evidence of the Mpemba effect. Surveying the literature, Burridge and Linden found that only Mpemba and Osborne, in their classic study, saw a Mpemba effect too pronounced to attribute to this kind of measurement error.

The findings “highlight how sensitive these experiments are even when you don’t include the freezing process,” said Burridge.

Strange Shortcuts

Yet a good number of researchers think the Mpemba effect can occur, at least under certain conditions. After all, Aristotle wrote in the fourth century BCE that “many people, when they want to cool water quickly, begin by putting it in the sun,” the benefits of which were presumably noticeable even before the invention of sensitive thermometers. School-age Mpemba was similarly able to observe the unsubtle difference between his frozen ice cream and his classmates’ slurry. Still, Burridge and Linden’s findings highlight a key reason why the Mpemba effect, real or not, might be so hard to pin down: Temperature varies throughout a cup of rapidly cooling water because the water is out of equilibrium, and physicists understand very little about out-of-equilibrium systems.

In equilibrium, a fluid in a bottle can be described by an equation with three parameters: its temperature, its volume and the number of molecules. Shove that bottle in a freezer, and all bets are off. The particles at the outer edge will be plunged into an icy environment while those deeper in will remain warm. Labels like temperature and pressure are no longer well defined but instead constantly fluctuate.

When Zhiyue Lu of the University of North Carolina read about the Mpemba effect in middle school, he snuck into an oil refinery in the Shandong province of China where his mother worked and used precision lab equipment to measure temperature as a function of time in a sample of water (he ended up supercooling the water without it freezing). Later, while studying nonequilibrium thermodynamics as a graduate student, he tried to reframe his approach to the Mpemba effect. “Is there any thermodynamic rule that will forbid the following: Something starting further away from the final equilibrium that would approach equilibrium faster than something starting from close?” he asked.

A pair of portraits of men looking at the camera.

Zhiyue Lu of the University of North Carolina (top) and Oren Raz of the Weizmann Institute of Science in Israel have shown that hot liquids may find “strange shortcuts” to their freezing points.

Robert Filcsik (top); Itai Belson / Weizmann Institute of Science

Zhiyue Lu of the University of North Carolina (left) and Oren Raz of the Weizmann Institute of Science in Israel have shown that hot liquids may find “strange shortcuts” to their freezing points.

Robert Filcsik (left); Itai Belson / Weizmann Institute of Science

Lu met Oren Raz , who now studies nonequilibrium statistical mechanics at the Weizmann Institute of Science in Israel, and they began developing a framework to investigate the Mpemba effect generally, not just in water. Their 2017 paper in the Proceedings of the National Academy of Sciences modeled the random dynamics of particles, showing that in principle there are nonequilibrium conditions under which the Mpemba effect and its inverse could occur. The abstract findings suggested that the components of a hotter system, by virtue of having more energy, are able to explore more possible configurations and therefore discover states that act as a sort of bypass, allowing the hot system to overtake a cool one as both dropped toward a colder final state.

“We all have this naive picture that says temperature should change monotonically,” said Raz. “You start at a high temperature, then a medium temperature, and go to a low temperature.” But for something driven out of equilibrium, “it’s not really true to say that the system has a temperature,” and “since that’s the case you can have strange shortcuts.”

The thought-provoking work drew the interest of others, including a Spanish group that began simulating what are known as granular fluids — collections of rigid particles that can flow like liquids, such as sand or seeds — and showed that these, too, can have Mpemba-like effects. Statistical physicist Marija Vucelja of the University of Virginia started wondering how common the phenomenon might be. “Is this like is a needle in a haystack, or could it be useful for optimal heating or cooling protocols?” she asked. In a 2019 study , she, Raz, and two co-authors found that the Mpemba effect could appear in a significant fraction of disordered materials, such as glass. While water is not such a system, the findings covered an enormous variety of possible materials.

To investigate whether these theoretical hunches had any real-world basis, Raz and Lu approached Bechhoefer, an experimentalist. “Literally, they kind of grabbed me after a talk and said, ‘Hey, we’ve got something we want you to hear about,’” Bechhoefer recalled.

Exploring the Landscape

The experimental setup Bechhoefer and his collaborator Avinash Kumar came up with offers a highly conceptual, stripped-down look at a collection of particles under the influence of different forces. A microscopic glass bead representing a particle is placed in a W-shaped “energy landscape,” created using lasers. The deeper of the two valleys in this landscape is a stable resting place. The shallower valley is a “metastable” state — a particle can fall into it but may eventually get knocked into the deeper well. The scientists submerged this landscape in water and used optical tweezers to position the glass bead within it 1,000 different times; collectively, the trials are equivalent to a system with 1,000 particles.

An initially “hot” system was one where the glass bead could be placed anywhere, since hotter systems have more energy and can therefore explore more of the landscape. In a “warm” system, the starting position was confined to a smaller area close to the valleys. During the cooling process, the glass bead first settled into one of the two wells, then spent a longer period jumping back and forth between them, buffeted by water molecules. Cooling was considered complete when the glass bead stabilized into spending specific amounts of time in each well, such as 20% of its time in the metastable one and 80% in the stable one. (These ratios depended on the water’s initial temperature and the valleys’ sizes.)

For certain initial conditions, the hot system took longer to settle into a final configuration than the warm system, matching our intuitions. But sometimes the particles in the hot system settled into the wells more quickly. When the experimental parameters were tuned just right, the hot system’s particles almost immediately found their final configuration, cooling exponentially faster than the warm system — a situation that Raz, Vucelja and colleagues had predicted and named the strong Mpemba effect. They reported the results in a 2020 Nature paper and published similar experiments showing the inverse Mpemba effect in PNAS earlier this year.

“The results are clear,” said Raúl Rica Alarcón of the University of Granada in Spain, who is working on independent experiments related to the Mpemba effect. “They show that a system that is farther away from the target can reach this target faster than another one that is closer to the target.”

A pair of portraits of men standing next to lab equipment.

Recent experiments with lasers and glass beads by Avinash Kumar (top) and John Bechhoefer of Simon Fraser University indicate that hot liquids can indeed relax to equilibrium faster than cold liquids.

Simon Fraser University (top); Dianne Mar-Nicolle

Recent experiments with lasers and glass beads by Avinash Kumar (left) and John Bechhoefer of Simon Fraser University indicate that hot liquids can indeed relax to equilibrium faster than cold liquids.

Simon Fraser University (left); Dianne Mar-Nicolle

Yet not everyone is entirely persuaded that the Mpemba effect has been demonstrated in any system. “I always read these experiments and I’m not impressed by the write-up,” said Burridge. “I never find a clear physical explanation, and I feel that leaves us with an interesting question as to whether Mpemba-like effects exist in a meaningful way.”

Bechhoefer’s trials appear to offer some insight into how the Mpemba effect could arise in systems with metastable states, but whether it is the only mechanism or how any particular substance undergoes such out-of-equilibrium heating or cooling is unknown.

Determining if the phenomenon occurs in water remains another open question. In April, Raz and his graduate student Roi Holtzman posted a paper showing that the Mpemba effect could happen through a related mechanism that Raz has previously described with Lu in systems that undergo a second-order phase transition, meaning that their solid and liquid forms can’t coexist at the same temperature. Water is not such a system (it has first-order phase transitions), but Bechhoefer described the work as gradually sneaking up on an answer for water.

If nothing else, the theoretical and experimental work on the Mpemba effect has started giving physicists a handhold into nonequilibrium systems that they otherwise lack. “Relaxation towards equilibrium is an important question that, frankly, we don’t have a good theory [for],” said Raz. Identifying which systems might behave in strange and counterintuitive ways “would give us a much better picture of how systems relax towards equilibrium.”

After igniting a decades-long controversy with his teenage interrogations, Mpemba himself went on to study wildlife management, becoming a principal game officer in Tanzania’s Ministry of Natural Resources and Tourism before retiring. According to Christine Osborne, the widow of Denis Osborne, Mpemba passed away around 2020. Science continues to spring from his insistence about the effect that bears his name. Osborne, discussing the results of their investigations together, took a lesson from the initial skepticism and dismissal that the schoolboy’s counterintuitive claim had faced: “It points to the danger of an authoritarian physics.”

Get highlights of the most important news delivered to your email inbox

Comment on this article

Quanta Magazine moderates comments to facilitate an informed, substantive, civil conversation. Abusive, profane, self-promotional, misleading, incoherent or off-topic comments will be rejected. Moderators are staffed during regular business hours (New York time) and can only accept comments written in English. 

Micrograph of a neuron showing aggregations of tau protein.

Next article

Original by Monwhea Jeng (Momo), Department of Physics, University of California, 1998.

Can hot water freeze faster than cold water?

Yes—a general explanation.

Hot water can in fact freeze faster than cold water for a wide range of experimental conditions.  This phenomenon is extremely counterintuitive, and surprising even to most scientists, but it is in fact real.  It has been seen and studied in numerous experiments .  Although this phenomenon has been known for centuries, and was described by Aristotle, Bacon, and Descartes [1–3] , it was not introduced to the modern scientific community until 1969, by a Tanzanian high school pupil named Mpemba.  Both the early scientific history of this effect, and the story of Mpemba's rediscovery of it, are interesting in their own right — Mpemba's story in particular providing a dramatic parable against making snap judgements about what is impossible.  This is described separately below.

The phenomenon that hot water may freeze faster than cold is often called the Mpemba effect.  Because, no doubt, most readers are extremely skeptical at this point, we should begin by stating precisely what we mean by the Mpemba effect.  We start with two containers of water, which are identical in shape, and which hold identical amounts of water.  The only difference between the two is that the water in one is at a higher (uniform) temperature than the water in the other.  Now we cool both containers, using the exact same cooling process for each container.  Under some conditions the initially warmer water will freeze first.  If this occurs, we have seen the Mpemba effect.  Of course, the initially warmer water will not freeze before the initially cooler water for all initial conditions.  If the hot water starts at 99.9°C, and the cold water at 0.01°C, then clearly under those circumstances, the initially cooler water will freeze first.  But under some conditions the initially warmer water will freeze first: if that happens, you have seen the Mpemba effect.  But you will not see the Mpemba effect for just any initial temperatures, container shapes, or cooling conditions.

This seems impossible, right?  Many sharp readers may have already come up with a common proof that the Mpemba effect is impossible.  The proof usually goes something like this.  Say that the initially cooler water starts at 30°C and takes 10 minutes to freeze, while the initially warmer water starts out at 70°C.  Now the initially warmer water has to spend some time cooling to get to get down to 30°C, and after that, it's going to take 10 more minutes to freeze.  So since the initially warmer water has to do everything that the initially cooler water has to do, plus a little more, it will take at least a little longer, right?  What can be wrong with this proof?

What's wrong with this proof is that it implicitly assumes that the water is characterized solely by a single number — its average temperature.  But if other factors besides the average temperature are important, then when the initially warmer water has cooled to an average temperature of 30°C, it may look very different than the initially cooler water (at a uniform 30°C) did at the start.  Why?  Because the water may have changed when it cooled down from a uniform 70°C to an average 30°C.  It could have less mass, less dissolved gas, or convection currents producing a non-uniform temperature distribution.  Or it could have changed the environment around the container in the refrigerator.  All four of these changes are conceivably important, and each will be considered separately below.  So the impossibility proof given above doesn't work.  And in fact the Mpemba effect has been observed in a number of controlled experiments [5,7–14]

It is still not known exactly why this happens.  A number of possible explanations for the effect have been proposed, but so far the experiments do not show clearly which, if any, of the proposed mechanisms is the most important one.  While you will often hear confident claims that X is the cause of the Mpemba effect, such claims are usually based on guesswork, or on looking at the evidence in only a few papers and ignoring the rest.  Of course, there is nothing wrong with informed theoretical guesswork or being selective in which experimental results you trust; the problem is that different people make different claims as to what X is.

Why hasn't modern science answered this seemingly simple question about cooling water? The main problem is that the time it takes water to freeze is highly sensitive to a number of details in the experimental setup, such as the shape and size of the container, the shape and size of the refrigeration unit, the gas and impurity content of the water, how the time of freezing is defined, and so on.  Because of this sensitivity, while experiments have generally agreed that the Mpemba effect occurs, they disagree over the conditions under which it occurs, and thus about why it occurs.  As Firth [7] wrote "There is a wealth of experimental variation in the problem so that any laboratory undertaking such investigations is guaranteed different results from all others."

Finally, supercooling may be important to the effect.  Supercooling occurs when the water freezes not at 0°C, but at some lower temperature.  One experiment [12] found that its initially hot water supercooled less than its initially cold water.  This would mean that the initially warmer water might freeze first because it would freeze at a higher temperature than the initially cooler water.  If true, this would not fully explain the Mpemba effect, because we would still need to explain why initially warmer water supercools less than initially cooler water.

In short, hot water does freeze sooner than cold water under a wide range of circumstances.  It is not impossible, and has been seen to occur in a number of experiments.  But despite claims often made by one source or another, there is no well-agreed explanation for how this phenomenon occurs.  Different mechanisms have been proposed, but the experimental evidence is inconclusive.  For those wishing to read more on the subject, Jearl Walker's article in Scientific American [13] is very readable and has suggestions on how to do home experiments on the Mpemba effect, while the articles by Auerbach [12] and Wojciechowski [14] are two of the more modern papers on the effect.

History of the Mpemba Effect

The fact that hot water freezes faster than cold has been known for many centuries.  The earliest reference to this phenomenon dates back to Aristotle in 300 B.C.  The phenomenon was later discussed in the medieval era, as European physicists struggled to come up with a theory of heat.  But by the 20th century the phenomenon was only known as common folklore, until it was reintroduced to the scientific community in 1969 by Mpemba, a Tanzanian high school pupil.  Since then, numerous experiments have confirmed the existence of the "Mpemba effect", but have not settled on any single explanation.

The earliest known reference to this phenomenon is by Aristotle, who wrote:

"The fact that water has previously been warmed contributes to its freezing quickly; for so it cools sooner.  Hence many people, when they want to cool hot water quickly, begin by putting it in the sun. . ." [1,4]

He wrote these words in support of a mistaken idea which he called antiperistasis.  Antiperistasis is defined as "the supposed increase in the intensity of a quality as a result of being surrounded by its contrary quality, for instance, the sudden heating of a warm body when surrounded by cold" [4] .

Medieval scientists believed in Aristotle's theory of antiperistasis, and also sought to explain it.  Not surprisingly, scientists in the 1400s had trouble explaining how it worked, and could not even decide whether (as Aristotle claimed in support of antiperistasis), human bodies and bodies of water were hotter in the winter than in the summer [4] .  Around 1461, the physicist Giovanni Marliani, in a debate over how objects cooled, said that he had confirmed that hot water froze faster than cold.  He said that he had taken four ounces of boiling water, and four ounces of non-heated water, placed them outside in similar containers on a cold winter day, and observed that the boiled water froze first.  Marliani was, however, unable to explain this occurrence [4] .

Later, in the 1600s, it was apparently common knowledge that hot water would freeze faster than cold.  In 1620 Bacon wrote "Water slightly warm is more easily frozen than quite cold" [2] , while a little later Descartes claimed "Experience shows that water that has been kept for a long time on the fire freezes sooner than other water" [3] .

In time, a modern theory of heat was developed, and the earlier observations of Aristotle, Marliani, and others were forgotten, perhaps because they seemed so contradictory to modern concepts of heat.  But it was still known as folklore among many non-scientists in Canada [11] , England [15–21] , the food processing industry [23] , and elsewhere.

It was not reintroduced to the scientific community until 1969, 500 years after Marliani's experiment, and more than two millennia after Aristotle's "Meteorologica I" [1] .  The story of its rediscovery by a Tanzanian high school pupil named Mpemba is written up in the New Scientist [4] .  The story provides a dramatic parable cautioning scientists and teachers against dismissing the observations of non-scientists and against making quick judgements about what is impossible.

In 1963, Mpemba was making ice cream at school, which he did by mixing boiling milk with sugar.  He was supposed to wait for the milk to cool before placing it the refrigerator, but in a rush to get scarce refrigerator space, put his milk in without cooling it.  To his surprise, he found that his hot milk froze into ice cream before that of other pupils.  He asked his physics teacher for an explanation, but was told that he must have been confused, since his observation was impossible.

Mpemba believed his teacher at the time.  But later that year he met a friend of his who made and sold ice cream in Tanga town.  His friend told Mpemba that when making ice cream, he put the hot liquids in the refrigerator to make them freeze faster.  Mpemba found that other ice cream sellers in Tanga had the same practice.

Later, when in high school, Mpemba learned Newton's law of cooling, that describes how hot bodies are supposed to cool (under certain simplifying assumptions).  Mpemba asked his teacher why hot milk froze before cold milk when he put them in the freezer.  The teacher answered that Mpemba must have been confused.  When Mpemba kept arguing, the teacher said "All I can say is that is Mpemba's physics and not the universal physics" and from then on, the teacher and the class would criticize Mpemba's mistakes in mathematics and physics by saying "That is Mpemba's mathematics" or "That is Mpemba's physics." But when Mpemba later tried the experiment with hot and cold water in the biology laboratory of his school, he again found that the hot water froze sooner.

Earlier, Dr Osborne, a professor of physics, had visited Mpemba's high school.  Mpemba had asked him to explain why hot water would freeze before cold water.  Dr Osborne said that he could not think of any explanation, but would try the experiment later.  When back in his laboratory, he asked a young technician to test Mpemba's claim.  The technician later reported that the hot water froze first, and said "But we'll keep on repeating the experiment until we get the right result." But repeated tests gave the same result, and in 1969 Mpemba and Osborne wrote up their results [5] .

In the same year, in one of the coincidences so common in science, Dr Kell independently wrote a paper on hot water freezing sooner than cold water.  Kell showed that if one assumed that the water cooled primarily by evaporation, and maintained a uniform temperature, the hot water would lose enough mass to freeze first [11] .  Kell thus argued that the phenomenon (then a common urban legend in Canada) was real and could be explained by evaporation.  But he was unaware of Osborne's experiments, which had measured the mass lost to evaporation and found it insufficient to explain the effect.  Subsequent experiments were done with water in a closed container, eliminating the effects of evaporation, and still found that the hot water froze first [14] .

Subsequent discussion of the effect has been inconclusive.  While quite a few experiments have replicated the effect [4,6–13] , there has been no consensus on what causes the effect.  The different possible explanations are discussed above .  The effect has repeatedly a topic of heated discussion in the "New Scientist", a popular science magazine.  The letters have revealed that the effect was known by laypeople around the world long before 1969.  Today, there is still no well-agreed explanation of the Mpemba effect.

Evaporation

One explanation of the effect is that as the hot water cools, it loses mass to evaporation.  With less mass, the liquid has to lose less heat to cool, and so it cools faster.  With this explanation, the hot water freezes first, but only because there's less of it to freeze.  Calculations done by Kell in 1969 [11] showed that if the water cooled solely by evaporation, and maintained a uniform temperature, the warmer water would freeze before the cooler water.

This explanation is solid, intuitive, and undoubtedly contributes to the Mpemba effect in most physical situations.  But many people have incorrectly assumed that it is therefore "the" explanation for the Mpemba effect.  That is, they assume that the only reason hot water can freeze faster than cold is because of evaporation, and that all experimental results can be explained by the calculations in Kell's article.  But the experiments currently do not bear this belief out.  While experiments show evaporation to be important [13] , they do not show that it is the only mechanism behind the Mpemba effect.  A number of experimenters have argued that evaporation alone is insufficient to explain their results [5,9,12] ; in particular, the original experiment by Mpemba and Osborne measured the mass lost to evaporation, and found it substantially less that the amount predicted by Kell's calculations [5,9] .  And most convincingly, an experiment by Wojciechowski observed the Mpemba effect in a closed container, where no mass was lost to evaporation.

Dissolved Gasses

Another explanation argues that the dissolved gas usually present in water is expelled from the initially hot water, and that this changes the properties of the water in some way that explains the effect.  It has been argued that the lack of dissolved gas may change the ability of the water to conduct heat, or change the amount of heat needed to freeze a unit mass of water, or change the freezing point of the water by some significant amount.  It is certainly true that hot water holds less dissolved gas than cold water, and that boiled water expels most dissolved gas.  The question is whether this can significantly affect the properties of water in a way that explains the Mpemba effect.  As far as I know, there is no theoretical work supporting this explanation for the Mpemba effect.

Indirect support can be found in two experiments that saw the Mpemba effect in normal water which held dissolved gasses, but failed to see it when using degassed water [10,14] .  But an attempt to measure the dependence of the enthalpy of freezing on the initial temperature and gas content of the water was inconclusive [14] .

One problem with this explanation is that many experiments pre-boiled both the initially hot and initially cold water, precisely to eliminate the effect of dissolved gasses, and yet they still saw the effect [5,13] .  Two somewhat unsystematic experiments found that varying the gas content of the water made no substantial difference to the Mpemba effect [9,12] .

It has also been proposed that the Mpemba effect can be explained by the fact that the temperature of the water becomes non-uniform.  As the water cools, temperature gradients and convection currents will develop.  For most temperatures, the density of water decreases as the temperature increases.  So over time, as water cools we will develop a "hot top" — the surface of the water will be warmer than the average temperature of the water, or the water at the bottom of the container.  If the water loses heat primarily through the surface, then this means that the water should lose heat faster than one would expect based just on looking at the average temperature of the water.  And for a given average temperature, the heat loss should be greater the more inhomogenous the temperature distribution is (that is, the greater the range of the temperatures seen as we go from the top to the bottom).

How does this explain the Mpemba effect?  Well, the initially hot water will cool rapidly, and quickly develop convection currents and so the temperature of the water will vary greatly from the top of the water to the bottom.  On the other hand, the initially cool water will have a slower rate of cooling, and will thus be slower to develop significant convection currents.  Thus, if we compare the initially hot water and initially cold water at the same average temperature, it seems reasonable to believe that the initially hot water will have greater convection currents, and thus have a faster rate of cooling.  To consider a concrete example, suppose that the initially hot water starts at 70°C, and the initially cold water starts at 30°C.  When the initially cold water is at an average 30°C, it is also a uniform 30°C.  But when the initially hot water reaches an average 30°C, the surface of the water is probably much warmer than 30°C, and it will thus lose heat faster than the initially cold water for the same average temperature.  Got that?  This explanation is pretty confusing, so you might want to go back and read the last two paragraphs again, paying careful attention to the difference between initial temperature, average temperature, and surface temperature.

At any rate, if the above argument is right, then when we plot the average temperature versus time for both the initially hot and initially cold water, then for some average temperatures the initially hot water will be cooling faster than the initially cold water.  So the cooling curve of the initially hot water will not simply reproduce the cooling curve of the initially cold water, but will drop faster when in the same temperature range.

This shows that the initially hot water goes faster, but of course it also has farther to go.  So whether it actually finishes first (that is, reaches 0°C first), is not clear from the above discussion.  To know which one finishes first would require theoretical modelling of the convection currents (hopefully for a range of container shapes and sizes), which has not been done.  So convection alone may be able to explain the Mpemba effect, but whether it actually does is not currently known.  Experiments on the Mpemba effect have often reported a "hot top" [5,8,10] , as we would expect.  Experiments have been done that looked at the convection currents of freezing water [27,28] , but their implications for the Mpemba effect are not entirely clear.

It should also be noted that the density of water reaches a maximum at four° C.  So below four°C, the density of water actually decreases with decreasing temperature, and we will get a "cold top." This makes the situation even more complicated.

Surroundings

The initially hot water may change the environment around it in some way that makes it cool faster later on.  One experiment reported significant changes in the data simply upon changing the size of the freezer that the container sat in [7] .  So conceivably it is important not just to know about the water and the container, but about the environment around it.

For example, one explanation for the Mpemba effect is that if the container is resting on a thin layer of frost, than the container holding the cold water will simply sit on the surface of the frost, while the container with the hot water will melt the frost, and then be sitting on the bottom of the freezer.  The hot water will then have better thermal contact with the cooling systems.  If the melted frost refreezes into an ice bridge between the freezer and the container, the thermal contact may be even better.

Obviously, even if this argument is true, it has fairly limited utility, since most scientific experiments are careful enough not to rest the container on a layer of frost in a freezer, but instead place the container on a thermal insulator, or in a cooling bath.  So while this proposed mechanism may or may not have some relevance to some home experiments, it's irrelevant for most published results.

Supercooling

Finally, supercooling may be important to the effect.  Supercooling occurs when water freezes not at 0°C, but at some lower temperature.  This happens because the statement that "water freezes at 0°C" is a statement about the lowest energy state of the water: at less than 0°C, the water molecules "want" to be arranged as an ice crystal.  This means that they will stop zooming around randomly as a liquid, and instead form a solid ice lattice.  But they don't know how to form themselves into an ice lattice, but need some small irregularity or nucleation site to tell them how to arrange themselves.  Sometimes, when water is cooled below 0°C, the molecules will not see a nucleation site for some time, and then water will cool below 0°C without freezing.  This happens quite often.  One experiment found that initially hot water would supercool only a little (say to about −2°C), while initially cold water would supercool more (to around −8°C) [12] .  If true, this could explain the Mpemba effect because the initially cold water would need to "do more work"; — that is, get colder — to freeze.

But this also cannot be considered "the" sole explanation of the Mpemba effect.  First of all, as far as I know, this result has not been independently confirmed.  The experiment described above [12] only had a limited number of trials, so the results found could have been a statistical fluke.

Second, even if the results are true, they do not fully explain the Mpemba effect, but replace one mystery with another.  Why should initially hot water supercool less than initially cold water?  After all, once the water has cooled to the lower temperature, one would generally expect that the water would not "remember" what temperature it used to be.  One explanation is that the initially hot water has less dissolved gas than the initially cold water, and that this affects its supercooling properties (see Dissolved Gasses for more on this).  The problem with this explanation is that one would expect that since the hot water has less dissolved gas, and thus fewer nucleation sites, it would supercool more, not less.  Another explanation is that when the initially hot water has cooled down to 0°C (or less), its temperature distribution throughout the container varies more than the initially cold water (see Convection for more on this).  Since temperature shear induces freezing [26] , the initially hot water supercools less, and thus freezes sooner.

Third, this explanation cannot work in all of the experiments, because many of the experimenters chose to look not at the time to form a complete block of ice, but the time for some part of the water to reach 0°C [7,10,13] (or perhaps the time for a thin layer of frost to form on the top [17] ).  While [12] says that it is only a "true Mpemba effect" if the hot water freezes entirely first, other papers have defined the Mpemba effect differently.  Since the precise time of supercooling is inherently unpredictable (see e.g. [26] ), many experiments have chosen to measure not the time for the sample to actually become ice, but the time for which the sample's equilibrium ground state is ice; that is, the time when the top of the sample reached 0°C [7,10,13] .  The supercooling argument does not apply to these experiments.

It's True: Hot Water Really Can Freeze Faster Than Cold Water

icy_hot

Hot water really can freeze faster than cold water, a new study finds. Sometimes. Under extremely specific conditions. With carefully chosen samples of water.

The Mpemba effect is named for a Tanzanian schoolboy, Erasto B. Mpemba, who noticed while making ice cream with his classmates that warm milk froze sooner than chilled milk. Mpemba and physicist Denis Osborne published a report of the phenomenon in Physics Education in 1969. Mpemba joined a distinguished group of people who had also noticed the effect: Aristotle, Francis Bacon and René Descartes had all made the same claim.

On the surface, the notion seems to defy reason. A container of hot water should take longer to turn into ice than a container of cold water, because the cold water has a head start in the race to zero degrees Celsius.

But under scientific scrutiny, the issue becomes murky. The new study doesn’t explain the phenomenon, but it does identify special conditions under which the Mpemba effect can be seen, if it truly exists.

“All in all, the work is a nice beginning, but not systematic enough to do more than confirm it can happen,” comments water expert David Auerbach, whose own experiments also suggest that the effect does occur.

Papers published over the last decade, including several by Auerbach, who performed his research while at the Max Planck Institute for Flow Research in Göttingen, Germany, have documented instances of hot water freezing faster than cold, but not reproducibly, says study author James Brownridge of State University of New York at Binghamton. “No one has been able to get reproducible results on command.”

That’s what Brownridge has done. One of his experiments, presented online, repeatedly froze a sample of hot water faster than a similar sample of cool water.

Note the word similar. In order for the experiment to work, the cool water had to be distilled, and the hot water had to come from the tap.

In the experiment, about two teaspoons of each sample were held in a copper device that completely surrounded the water, preventing evaporation and setting reasonably even temperatures. Freezing was official when sensors picked up an electrical signal created by ice formation.

Brownridge heated the tap water to about 100° C, while the distilled water was cooled to 25° C or lower. When both samples were put into the freezer, the hot water froze before the cold water. Brownridge then thawed the samples and repeated the experiment 27 times. Each time, the hot tap water froze first.

The experiment worked because the two types of water have different freezing points, Brownridge says. Differences in the shape, location and composition of impurities can all cause water’s freezing temperature — which in many cases is below zero degrees C — to vary widely. With a higher freezing point, the tap water had an edge that outweighed the distilled water’s lower temperature.

Because the experiment didn’t compare two identical samples of water, the mystery of the Mpemba effect is not really solved. “I’m not arrogant enough to say I’ve solved this,” Brownridge says. But he has set some guidelines about when the effect can be seen.

Physical chemist Christoph Salzmann of the University of Durham in England says he’s not convinced the Mpemba effect really exists, because there are innumerable things that influence the timing of freezing, making it impossible to completely control.

Predicting how long it will take for a water sample to crystallize “is a bit like trying to predict when the next earthquake or crash of the stock market will happen,” he says. “I would not want to say that the Mpemba effect does not exist. But I have still not been convinced of its existence.”

Image: Kenn Wilson /flickr

The Snowflake Attack May Be Turning Into One of the Largest Data Breaches Ever

By Matt Burgess

The End of El Niño Might Make the Weather Even More Extreme

By Sachi Mulkey

Microsoft Will Switch Off Recall by Default After Security Backlash

By Andy Greenberg

The Lords of Silicon Valley Are Thrilled to Present a ‘Handheld Iron Dome’

By Matthew Gault

Zombie Fire Season Is Here in the Arctic

Sebastian Wieczorek

NASA’s Quest to Touch the Sun

Thomas Zurbuchen

What’s Up With These Crazy Northern Lights?

Rhett Allain

The World Is Ignoring the Other Deadly Kind of Carbon

Jon Brodkin, Ars Technica

Sachi Mulkey

Does String Theory Actually Describe the World? AI May Be Able to Tell

Charlie Wood

How Much Energy Would It Take to Pull Carbon Dioxide out of the Air?

Life's Little Mysteries

Does Hot Water Freeze Faster Than Cold Water?

ice spike, ices freezes, hot water, cold water

Determining whether or not hot water can freeze faster than cold water may seem like a no-brainer. After all, water freezes at 0 degrees Celsius. And wouldn’t water hot enough to kill E. coli bacteria (about 120 degrees Fahrenheit or 50 degrees Celsius) take a longer path than cooler water at a fall New England beach (about 60 degrees Fahrenheit or 15 degrees Celsius) towards a frigid future as ice? While a logical assumption, it turns out that hot water can freeze before cooler water under certain conditions.

This apparent quirk of nature is the "Mpemba effect," named after the Tanzanian high school student, Erasto Mpemba, who first observed it in 1963. The Mpemba effect occurs when two bodies of water with different temperatures are exposed to the same subzero surroundings and the hotter water freezes first. Mpemba’s observations confirmed the hunches of some of history’s most revered thinkers, such as Aristotle, Rene Descartes and Francis Bacon, who also thought that hot water froze faster than cold water.

Evaporation is the strongest candidate to explain the Mpemba effect. As hot water placed in an open container begins to cool, the overall mass decreases as some of the water evaporates. With less water to freeze, the process can take less time. But this doesn’t always work, especially when using closed containers that prevent evaporated water from escaping.

And evaporation may not be the only reason the water can freeze more quickly. There may be less dissolved gas in the warmer water, which can reduce its ability to conduct heat, allowing it to cool faster. However, Polish physicists in the 1980s were unable to conclusively demonstrate this relationship.

— Why rain gives off that fresh, earthy smell 

— What's the largest waterfall in the world?  

— What's this spike doing on my ice cube?  

A non-uniform temperature distribution in the water may also explain the Mpemba effect. Hot water rises to the top of a container before it escapes, displacing the cold water beneath it and creating a "hot top." This movement of hot water up and cold water down is called a convection current. These currents are a popular form of heat transfer in liquids and gases, occurring in the ocean and also in radiators that warm a chilly room. With the cooler water at the bottom, this uneven temperature distribution creates convection currents that accelerate the cooling process. Even with more ground to cover to freeze, the temperature of the hotter water can drop at a faster rate than the cooler water.

So the next time you refill your ice cube tray, try using warmer water. You might have ice cubes to cool your drink even sooner.

This answer is provided by Scienceline , a project of New York University's Science, Health and Environmental Reporting Program.

Sign up for the Live Science daily newsletter now

Get the world’s most fascinating discoveries delivered straight to your inbox.

Originally published on Live Science.

Rare-earth elements could be hidden inside coal mines

100-foot 'walking tree' in New Zealand looks like an Ent from Lord of the Rings — and is the lone survivor of a lost forest

Restless legs syndrome tied to 140 'hotspots' in the genome

Most Popular

  • 2 2,000-year-old rock art, including nearly 140-foot-long snake, may mark ancient territories in Colombia, Venezuela
  • 3 The brain can store nearly 10 times more data than previously thought, study confirms
  • 4 'She is so old': One-eyed wolf in Yellowstone defies odds by having 10th litter of pups in 11 years
  • 5 100-foot 'walking tree' in New Zealand looks like an Ent from Lord of the Rings — and is the lone survivor of a lost forest
  • 2 100-foot 'walking tree' in New Zealand looks like an Ent from Lord of the Rings — and is the lone survivor of a lost forest
  • 3 A 'new star' could appear in the sky any night now. Here's how to see the Blaze Star ignite.
  • 4 Black hole 'morsels' could finally prove Stephen Hawking's famous theory right
  • 5 Save $400 on Unistellar's new smart binoculars during their early bird Kickstarter

does hot water or cold water freeze faster experiment

Is It True Hot Water Freezes Faster Than Cold?

Understand the Mpemba Effect

  • Chemistry In Everyday Life
  • Chemical Laws
  • Periodic Table
  • Projects & Experiments
  • Scientific Method
  • Biochemistry
  • Physical Chemistry
  • Medical Chemistry
  • Famous Chemists
  • Activities for Kids
  • Abbreviations & Acronyms
  • Weather & Climate
  • Ph.D., Biomedical Sciences, University of Tennessee at Knoxville
  • B.A., Physics and Mathematics, Hastings College

Yes, hot water can freeze faster than cold water. However, it does not always happen, nor has science explained exactly why it can happen.

Key Takeaways: Water Temperature and Rate of Freezing

  • Sometimes hot water freezes more quickly than cold water. This is called the Mpemba effect after the student who observed it.
  • Factors that may cause hot water to freeze faster include evaporative cooling, less chance of supercooling, low concentration of dissolved gases, and convection.
  • Whether hot or cold water freezes more quickly depends on the specific conditions.

The Mpemba Effect

Although Aristotle, Bacon, and Descartes all described hot water freezing faster than cold water, the notion was mostly resisted until the 1960's when a high school student named Mpemba noticed that hot ice cream mix, when placed into the freezer, would freeze before ice cream mix that had been cooled to room temperature before being placed in the freezer. Mpemba repeated his experiment with water rather than ice cream mixture and found the same result: the hot water froze more quickly than the cooler water. When Mpemba asked his physics teacher to explain the observations, the teacher told Mpemba his data must be in error, because the phenomenon was impossible.

Mpemba asked a visiting physics professor, Dr. Osborne, the same question. This professor replied that he did not know, but he would test the experiment. Dr. Osborne had a lab tech perform Mpemba's test. The lab tech reported that he had duplicated Mpemba's result, "But we'll keep on repeating the experiment until we get the right result." (Um... yeah... that would be an example of poor science.) Well, the data was the data, so when the experiment was repeated, it continued to yield the same result. In 1969 Osborne and Mpemba published the results of their research. Now the phenomenon in which hot water may freeze faster than cold water is sometimes called the Mpemba Effect.

Why Hot Water Sometimes Freezes Faster Than Cold Water

There is no definitive explanation for why hot water may freeze faster than cold water. Different mechanisms come into play, depending on the conditions. The main factors appear to be:

  • Evaporation : More hot water will evaporate than cold water, thus reducing the amount of water remaining to be frozen. Mass measurements lead us to believe this is an important factor when chilling water in open containers, though it isn't the mechanism that explains how the Mpemba Effect occurs in closed containers.
  • Supercooling : Hot water tends to experience less of a supercooling effect than cold water. When was supercools, it can remain a liquid until it is disturbed, even well below its normal freezing temperature. Water that is not supercooled is more likely to become solid when it reaches the freezing point of water .
  • Convection : Water develops convection currents as it cools. Water density usually decreases as temperature increases, so a container of cooling water typically is warmer on top than on the bottom. If we assume water loses most of its heat across its surface (which may or may not be true, depending on the conditions), then water with a hotter top would lose its heat and freeze faster than water with a cooler top.
  • Dissolved Gases : Hot water has less capacity to hold dissolved gases than cold water, which may affect its rate of freezing.
  • Effect of the Surroundings : The difference between the initial temperatures of two containers of water may have an effect on the surroundings that could influence the rate of cooling. One example would be warm water melting a pre-existing layer of frost, permitting a better cooling rate.

Test It Yourself

Now, don't take my word for this! If you are doubtful that hot water sometimes freezes more quickly than cold water, test it for yourself. Be aware the Mpemba Effect will not be seen for all experimental conditions, so you may need to play around with the size of the water sample and the cooling water (or try making ice cream in your freezer, if you'll accept that as a demonstration of the effect).

  • Burridge, Henry C.; Linden, Paul F. (2016). "Questioning the Mpemba effect: Hot water does not cool more quickly than cold". Scientific Reports . 6: 37665. doi: 10.1038/srep37665
  • Tao, Yunwen; Zou, Wenli; Jia, Junteng; Li, Wei; Cremer, Dieter (2017). "Different Ways of Hydrogen Bonding in Water - Why Does Warm Water Freeze Faster than Cold Water?". Journal of Chemical Theory and Computation . 13 (1): 55–76. doi: 10.1021/acs.jctc.6b00735
  • 10 Cool Chemistry Experiments
  • Weird Water Facts
  • Which Is Faster: Melting Ice in Water or Air?
  • Fun and Interesting Chemistry Facts
  • Two Methods for Supercooling Water
  • Bubble Life & Temperature
  • Does Blowing on Hot Food Really Make It Cooler?
  • Why Does Salt Melt Ice? Science of How It Works
  • Did You Know? Fun Chemistry Facts
  • Melting Point Vs. Freezing Point
  • How to Make Ice Spikes in your Freezer
  • Why Vodka Doesn't Freeze in Most Home Freezers
  • Make a Slushy Instantly With Soda and Supercooling
  • Thermal Inversion
  • Solidification Definition and Examples in Chemistry
  • When to Cite a Source in a Paper

It’s a wonderful world — and universe — out there.

Come explore with us!  

Science News Explores

Here’s how hot water might freeze faster than cold.

Properties of hydrogen bonds may explain the effect — if it even exists

frozen water

Hot water might freeze faster than cold water under certain conditions, some scientists say. Properties of hydrogen bonds could explain how this happens. But not all scientists agree that the effect is even real.

Harald Hoyer/Wikimedia Commons ( CC-BY-SA 2.0 )

Share this:

  • Google Classroom

By Emily Conover

January 25, 2017 at 7:10 am

Cold water should freeze faster than hot water. Right? It seems logical. But some experiments have suggested that under the right conditions, hot water can freeze faster than cold. Now chemists offer a new explanation for how this might happen.

What they don’t do, however, is confirm that it actually does occur.

The speedier freezing of hot water is known as the Mpemba effect. If it happens, it would be only under certain conditions. And those conditions would involve the bonds that link up neighboring water molecules. A team of chemists describe these potential unusual freezing properties in a paper published online December 6 in the  Journal of Chemical Theory and Computation .

Their paper has not, however, convinced everyone. Some skeptics argue that the effect just isn’t real.

People have described quick-freezing of hot water since the early days of science. Aristotle was a Greek philosopher and scientist. He lived in the 300s B.C. Back then, he reported observing hot water freezing faster than cold water. Fast forward to the 1960s. That’s when a student from the East African nation of Tanzania, Erasto Mpemba, noticed something strange, too. He  claimed that his ice cream turned into a solid faster when it was put into the freezer steaming hot. Scientists soon named the quick-freezing hot-water phenomenon for Mpemba.

No one is sure what might cause such an effect, although plenty of researchers have guessed at explanations. One is related to evaporation. That’s the transition of a liquid to a gas. Another has to do with convection currents. Convection occurs when some hotter material in a fluid or gas rises and colder material sinks. Yet another explanation suggests that that gasses or other impurities in water might alter its freezing rate. Still, none of these explanations has won over the general scientific community.

Explainer: What is a computer model?

Now along comes Dieter Cremer of Southern Methodist University in Dallas, Texas. This theoretical chemist has used  computer models to simulate to actions of atoms and molecules. In a new paper, he and his colleagues propose that chemical linkages — bonds — between water molecules could help explain any Mpemba effect.

Unusual links between the water molecules?

Hydrogen bonds are links that can form between hydrogen atoms of one molecule and the oxygen atom of a neighboring water molecule. Cremer’s group studied the strengths of these bonds. To do that they used a computer program that simulated how water molecules would cluster.

As water warms, Cremer notes, “We see that hydrogen bonds change.” The strength of these bonds can differ based on how the nearby water molecules are arranged. In simulations of cold water, both weak and strong hydrogen bonds develop. But at higher temperatures, the model predicts that a bigger share of the hydrogen bonds will be strong. It seems, Cremer says, “The weaker ones are broken to a large extent.”

His team realized that its new understanding of hydrogen bonds might explain the Mpemba effect. As water is warmed, weaker bonds would break. This would cause big clusters of these linked molecules to fragment into smaller clusters. Those fragments might realign to form tiny ice crystals. They could then serve as starting points for the bulk freezing to proceed. For cold water to rearrange in this way, weak hydrogen bonds would first have to break.

“The analysis in the paper is very well done,” says William Goddard. He is a chemist at the California Institute of Technology in Pasadena. But, he adds: “The big question is, ‘Does it actually relate directly to the Mpemba effect?’”

Cremer’s group noted an effect that might trigger the phenomenon, he says. But those scientists didn’t simulate the actual freezing process. They didn’t demonstrate that it happens faster when the new hydrogen bonding insights are included. Simply put, Goddard explains, the new study “doesn’t actually make the final connection.”

Somel scientists have a bigger concern with the new study. Among them is Jonathan Katz. A physicist, he works at Washington University in St. Louis. The idea that warm water might freeze faster than cold water “just makes absolutely no sense,” he says. In Mpemba experiments, the water freezes over a period of minutes or hours. As the temperature drops over that period of time, weak hydrogen bonds would reform and molecules would rearrange, Katz argues.

Other researchers also are debating whether the Mpemba effect exists. Scientists have struggled to produce the effect in a repeatable way. For example, one group of scientists measured the time for hot and cold samples of water to cool to zero degrees Celsius (32 degrees Fahrenheit). “No matter what we did, we could not observe anything akin to the Mpemba effect,” says Henry Burridge. He is an engineer at Imperial College London in England. He and colleagues published their results November 24 in  Scientific Reports .

But their study “excluded a very important aspect of the phenomenon,” says Nikola Bregović. He is a chemist at the University of Zagreb in Croatia. He says Burridge’s study observed only the time to reach the temperature at which water freezes. It did not observe the initiation of freezing itself. And, he points out, the process of freezing is complex and hard to control. That is one reason the Mpemba effect has been so hard to investigate. But, he adds, “I am still convinced that hot water can freeze more quickly than cold water.”

More Stories from Science News Explores on Chemistry

a composite image showing a black metal rectangle glued to chicken, a tomato, and an onion

A bit of electricity can glue hard metals to soft materials

Infrared composite image of Titan, Saturn's largest moon. The moon appears blue-green in the image with dark splotches across its surface.

Comets may be the source of sandy dunes on Saturn’s largest moon

Close-up of a young woman's sweat-stained underarm area.

Here’s why teens’ body odor can be especially strong

tiny bits of colored plastic scattered in soil

To limit pollution, new recipe makes plastic a treat for microbes

Against a night-black background, a hawkmoth hovers over a paper filter cone that is designed to mimic a night-blooming flower. The hawkmoth's long proboscis is reaching into the center of the cone.

Air pollution can make it harder for pollinators to find flowers

several pieces of denim dyed different shades of blue are fanned out on top of each other

Turning jeans blue with sunlight might help the environment

a poison dart frog with a black-spotted, golden body and black feet sitting on a rock

At last: How poison dart frogs ship defense toxins to their skin

an illustration of graphene shows a net-like sheet of atoms connected in a honeycomb pattern

Let’s learn about graphene

Stack Exchange Network

Stack Exchange network consists of 183 Q&A communities including Stack Overflow , the largest, most trusted online community for developers to learn, share their knowledge, and build their careers.

Q&A for work

Connect and share knowledge within a single location that is structured and easy to search.

Does hot water freeze faster than cold water?

Is it true that hot water freezes faster than cold water and if so, what practical applications have there been found for this phenomenon?

Suma's user avatar

  • 2 I know it does because Peggy Knapp did it on Newton's Apple . That was a great program. –  mmyers May 19, 2011 at 15:54
  • 3 I tried this by putting two glasses with the same volume of water in the freezer, one glass had room temperature water the other had hotter, recently boiled water. My outcome was that the cold water froze first as you'd expect. It is a well documented effect though and I'd be interested to know why my experiment gave a negative result. –  Stephen Paulger May 20, 2011 at 14:31
  • 3 If evaporation is a key factor, then the surface area will be important for the experiment. Two glasses of water in a freezer will not yield the same result as two shallow amounts of water spread over a square foot each. The hot water will evaporate much faster when it is very shallow and spread out. It has less total mass to retain heat and a lot more surface area to cool it and evaporate it. The hot water will evaporate reducing mass and will then freeze faster than the cold water. The relative humidity in the air will also be a variable to consider. –  user2952 May 20, 2011 at 14:51
  • 3 I once heard a rebuttal to this by claiming that hot water must become cold water before it could become frozen water. I find that a good example of completely missing the point. –  MrHen May 20, 2011 at 19:42
  • Related. physics.stackexchange.com/questions/122742/… –  Takahiro Waki Jan 17, 2018 at 6:01

4 Answers 4

In certain settings, cold water freezers slower than hot water. This is called the Mpemba effect .

The Mpemba effect is the observation that warmer water sometimes freezes faster than colder water. Although the observation has been verified, there is no single scientific explanation for the effect.

Can hot water freeze faster than cold water? , Monwhea Jeng, University of California, 1998

Hot water can in fact freeze faster than cold water for a wide range of experimental conditions. This phenomenon is extremely counterintuitive, and surprising even to most scientists, but it is in fact real. It has been seen and studied in numerous experiments. While this phenomenon has been known for centuries, and was described by Aristotle, Bacon, and Descartes [1—3], it was not introduced to the modern scientific community until 1969, by a Tanzanian high school student named Mpemba.

Some suggested reasons given in the paper:

Evaporation — As the initially warmer water cools to the initial temperature of the initially cooler water, it may lose significant amounts of water to evaporation. The reduced mass will make it easier for the water to cool and freeze. Then the initially warmer water can freeze before the initially cooler water, but will make less ice. [...] Dissolved Gasses — Hot water can hold less dissolved gas than cold water, and large amounts of gas escape upon boiling. So the initially warmer water may have less dissolved gas than the initially cooler water. [...]

Community's user avatar

  • 2 I think it is worth giving at least some of the suggested reasons here (e.g. more evaporation of the hot water means less water to freeze). –  Oddthinking ♦ May 19, 2011 at 7:07
  • 7 Wouldn't these theories, particularly the evaporation theory easily be tested??? For example measure how much ice is in each sample after the experiment??? –  kralco626 May 19, 2011 at 10:33
  • 5 @mplungjan That's actually a different phenomenon called supercooling, where a liquid is cooled to below it's freezing point without it actually freezing (because it lacks a nucleus point from where the freezing should start), and freezes instantly upon applying a shock, or something that makes it non-homogenous. Check out wikipedia for more info: en.wikipedia.org/wiki/Supercooling –  Andrei Fierbinteanu May 19, 2011 at 11:15
  • 3 One plausible explanation is that the warmer water has a stronger convection current when cooling. The angular momentum of the convection current sustains the current after dropping to a lower temperature, and so the water cools throughout more rapidly. –  Richard Gadsden Jul 1, 2012 at 17:42
  • 5 Interesting answer is at MIT web: Does hot water freeze faster than cold water? Their conclusion is no . –  Palec Nov 25, 2013 at 3:58

This was, actually, my 6 th 5 th grade Science Fair experiment. :)

And I'd never heard of this effect before; it was a random experiment I thought of and tried.

My answer: it depends on what you mean by "freeze" .

Cold water starts freezing sooner (entering 0 degrees C), but hot water finishes freezing sooner (leaving 0 degrees C). I measured this with a digital thermometer.

No idea why, but I'm darn sure my experiment was accurate.

I found the data!

Page 1

I blurred out the years to avoid carbon dating myself. ;)

user541686's user avatar

  • 3 This post does not cite any references or sources. Please help improve this article by adding citations to reliable sources. Unsourced material may be challenged and removed. –  Larian LeQuella Jan 13, 2012 at 21:17
  • 15 @LarianLeQuella: Does original research count? –  user541686 Jan 13, 2012 at 21:20
  • 57 Does "publication" and "peer review" at a science fair count for nothing these days? ;p BTW, did you win anything? –  Dan Moulding Mar 1, 2012 at 12:51
  • 9 Bravo! Anyway, the fact that you, at age ten, had access to a computer for analysis already partially carbon dates you. :) –  JasonSmith Jan 7, 2015 at 14:40
  • 12 Yeah, I guess I silicon-dated myself :) –  user541686 Jan 30, 2017 at 0:13

A new paper on this phenomenon has been published recently. It offers yet another explanation and has even caught the attention of popular media.

doi:10.1038/srep03005

arXiv:1310.6514v2 [physics.chem-ph]

They say the interaction between the hydrogen bonds and the stronger bonds that hold the hydrogen and oxygen atoms in each molecule together, known as covalent bonds, is what causes the effect. Normally when a liquid is heated, the covalent bonds between atoms stretch and store energy. The scientists argue that in water, the hydrogen bonds produce an unusual effect that causes the covalent bonds to shorten and store energy when heated. This they say leads to the bonds to release their energy in an exponential way compared to the initial amount stored when they are cooled in a freezer. So hot water will lose more energy faster than cool water. Dr Changqing said: “Heating stores energy by shortening and stiffening the H-O covalent bond. “Cooling in a refrigerator, the H-O bond releases its energy at a rate that depends exponentially on the initially stored energy, and therefore, Mpemba effect happens.” The Royal Society of Chemistry received more than 22,000 responses to its call for a solution to the Mpemba effect and it is still receiving theories despite the competition closing a year ago.

Quoted from Telegraph.co.uk .

Glorfindel's user avatar

It is true, in proper circumstances.

The Scientific explanation for that relates to the fact the freezing temperature may increase with the pressure.

The Mpemba effect is about freezing hot samples faster than cold which may not represent a substantial difference with small pressure variations but phenomenons like supercooling and superheating do have practical applications such as better preservation of organs in medical refrigerators and superconductivity in electrical devices.

You can find more about this in: The Mpemba effect: why hot water can sometimes freeze faster than cold .

Ken Graham's user avatar

  • 6 The freezing temperature actually decreases when the pressure increases... (see the phase diagram of water : 1.bp.blogspot.com/_Ukz5Qzczfbc/TVEUPJxtDfI/AAAAAAAAB54/… ) –  Jules Olléon May 20, 2011 at 14:21

You must log in to answer this question.

Not the answer you're looking for browse other questions tagged physics water ., hot network questions.

  • Inductance after core saturation
  • Which ability checks are rolled for a shove attack?
  • Why "Power & battery" stuck at spinning circle for 4 hours?
  • Datasheet recommends driving relay in an uncommon configuration
  • is it correct to say "push the table by its far edge"?
  • Estimating Probability Density for Sample
  • Prove that "max independent set is larger than max clique" is NP-Hard
  • Where do UBUNTU_CODENAME and / or VERSION_CODENAME come from?
  • What is the frequentist's Bayesian prior for a coin with unknown bias
  • Understanding the Amanda Knox, Guilty Verdict for Slander
  • What scientific evidence there is that keeping cooked meat at room temperature is unsafe past two hours?
  • How to create an enumerate environment with Case 1, Case 2,
  • Is 1.5 hours enough for flight transfer in Frankfurt?
  • What should I get paid for if I can't work due to circumstances outside of my control?
  • Is it rational for heterosexuals to be proud that they were born heterosexual?
  • Build the first 6 letters of an Italian codice fiscale (tax identification number)
  • Could a 200m diameter asteroid be put into a graveyard orbit and not be noticed by people on the ground?
  • Are there any jobs that are forbidden by law to convicted felons?
  • Is obeying the parallelogram law of vector addition sufficient to make a physical quantity qualify as a vector?
  • NES Emulator in C
  • StreamPlot does not give me the streamlines I ask for
  • Quick release inside of thru axle?
  • Calculation of centrifugal liquid propellant injectors
  • My vehicle shut off in traffic and will not turn on

does hot water or cold water freeze faster experiment

Stack Exchange Network

Stack Exchange network consists of 183 Q&A communities including Stack Overflow , the largest, most trusted online community for developers to learn, share their knowledge, and build their careers.

Q&A for work

Connect and share knowledge within a single location that is structured and easy to search.

Hot water freezing faster than cold water

This question has puzzled me for a long time. There is already a question like this on Physics.SE. John's answer to the question seems quite satisfying. But when I googled the cause I found this and this explanation. They maybe wrong but I think How Stuff Works is a reliable source.

And here's the original paper.

I am quite confused now reading the different explanations. Can anyone please shed some light on the issue?

  • thermodynamics
  • temperature
  • atomic-physics

Community's user avatar

  • $\begingroup$ Related mathoverflow question: mathoverflow.net/q/153669/13917 $\endgroup$ –  Qmechanic ♦ Jul 2, 2014 at 13:37
  • $\begingroup$ I see that it's the same paper. But has this explanation been confirmed or is just a hypothesis? $\endgroup$ –  Yashbhatt Jul 2, 2014 at 13:41
  • $\begingroup$ Maybe the reason for contradictory explanations is that we do not really know yet. $\endgroup$ –  Daniel Mahler Jul 22, 2014 at 6:39
  • $\begingroup$ @DanielMahler That is what the question is all about. $\endgroup$ –  Yashbhatt Jul 22, 2014 at 7:00
  • $\begingroup$ @Yashbhatt The answers here & in the links tend sound like we do know the answer, even if they may disagree on what it is. The question is asking for a definite/canonical answer. I am suggesting that such an answer may not be possible at present. $\endgroup$ –  Daniel Mahler Jul 22, 2014 at 7:16

7 Answers 7

To start with, "water freezes faster when it starts out hot" is not terribly precise . There are lots of different experiments you could try, over a huge range of initial conditions, that could all give different results. Wikipedia quotes an article Hot Water Can Freeze Faster Than Cold by Jeng which reviews approaches to the problem up to 2006 and proposes a more precise definition of the problem:

There exists a set of initial parameters, and a pair of temperatures, such that given two bodies of water identical in these parameters, and differing only in initial uniform temperatures, the hot one will freeze sooner.

However, even that definition still has problems, which Jeng recognizes: first, there's the question of what "freeze" means (some ice forms, or the water freezes solid all the way through); second, the hypothesis is completely unfalsifiable. Even if you restrict the hypothesis to the range of conditions reasonably attainable in everyday life, to explain why the effect is so frequently noted anecdotally, there's literally an infinite number of possible experimental conditions to test, and you can always claim that the correct conditions just haven't been tested yet.

So, the fact that the internet is awash in a variety of different explanations makes perfect sense: there really are a bunch of different reasons why initially hotter water may freeze faster than initially colder water, depending on the precise situation and the definition of "freeze" that you use.

The paper you link to, O:H-O Bond Anomalous Relaxation Resolving Mpemba Paradox by Zhang et al., with results echoed by the HowStuffWorks video, attempts to solve the problem for a very specific sub-hypothesis. They eliminate the problem of defining freezing by treating freezing as a proxy for cooling in general, and directly measuring cooling rates instead. That experimental design implicitly eliminates one internet-provided explanation right off the bat: it can't possibly be supercooling, because whether the water supercools or solidifies when it gets to freezing temperature is an entirely different question from how quickly it cools to a temperature where it could freeze.

They also further constrain the problem by looking for explanations that cannot apply to any other liquid. After all, the Mpemba effect is about why hot water freezes faster; nobody is reporting anomalous freezing of, say, hot alcohol. That might just be because people freeze water a lot, and we don't tend to work with a lot of other exotic chemicals in day-to-day life, but choosing to focus on that restriction makes the problem more well-defined, and again implicitly rules out a lot of potential explanations ahead of time- i.e., it can't have anything to do with evaporation (because lots of liquids undergo evaporative cooling, and that's cheating anyway 'cause it changes the mass of the liquid under consideration) or conduction coupling to the freezer shelf (because that has nothing to do with the physical properties of the liquid, and everything to do with an uncontrolled experimental environment, as explained by John Rennie .

So, there really isn't just one answer to "why does hot water freeze faster than cold water", because the question is ill-posed. If you give someone a specific experimental set-up, then you can get a specific answer, and there are a lot of different answers for different set-ups. But, if you want to know "why does initially-hotter water cool faster through a range of lower temperatures than water that started out at those lower temperatures, while no other known liquid appears to behave this way" (thus contributing to it freezing first if it doesn't supercool), Zhang has your answer, and it's because of the weird interplay between water's intra- and inter-molecular bond energies. As far as I can tell, that paper has not yet been replicated, so you may consider it unconfirmed, but it's a pretty well-reasoned explanation for a very specific question, which is probably an influencing factor in a lot of other cooling-down-hot-water situations. There is a follow-up article, Mpemba Paradox Revisited -- Numerical Reinforcement , which provides additional simulation evidence for the bond-energy explanation, but it can't really be considered independent confirmation because it's by the same four authors.

Logan R. Kearsley's user avatar

  • $\begingroup$ Thanks for such a nice answer. That clears a lot of confusion. But I din't get what you are trying to say int the 4th Para. do you mean something like hot water cools faster but doesn't freeze faster? $\endgroup$ –  Yashbhatt Jul 18, 2014 at 16:53
  • $\begingroup$ I'm saying "freezing" isn't very well defined, so Zhang measured something else. However you define freezing, we can at least agree it requires water to be cold, so how fast water cools is a good stand-in for the intuitive notion of how fast it freezes. Zhang determined that hot water can cool down faster, and therefore will probably freeze sooner, once you've controlled for all the different kinds of freezing (whether it supercools instead of solidifying, whether you count any crystal formation or solid-right-through, etc.), but "freezing" isn't part of the measured effect. Does that help? $\endgroup$ –  Logan R. Kearsley Jul 18, 2014 at 17:02
  • 3 $\begingroup$ That's Zhang article is incredibly sketchy "science." $\endgroup$ –  user10851 Jul 18, 2014 at 17:03
  • $\begingroup$ @ChrisWhite Could you be more... specific? What do you find wrong with it? In any case, the accuracy / believability of the Zhang article is kind of irrelevant- whether or not their explanation is correct, the main point is that the Mpemba effect is ill-defined, and thus the correct explanation in a specific circumstance legitimately varies quite a bit, and the Zhang article linked by the OP represents an attempt to explain a much more constrained, much more well-defined version of the problem. $\endgroup$ –  Logan R. Kearsley Jul 18, 2014 at 17:08
  • $\begingroup$ @LoganR.Kearsley So, is there no way to check if Zhang is correct as the all the parameters cannot be adjusted for different instances of the experiment? $\endgroup$ –  Yashbhatt Jul 18, 2014 at 17:25

This happens due to cooling affect of evapourisation.

As you must be knowing, the temperature of the lquid is a factor of evapourisation. So as the temperature of hot water is more, the rate of evapourisation is also more. Now this is where thwe cooling effect of evapourisation takes place. As the water evapourates, it takes away some heat thus cooling the hot water . so as a equation we can write it as: Rate of cooling of hot water = cooling effect of evapourisation + cooling effect of freezer Rate of cooling of cold water = cooling effect of freezer

Thus we can state that cooling effect of hot water is more than cooling effect of cold water

There is also one more thing that affects the rate of cooling. As the differences between the temperature of surroundings and the temperature of liquid is more in hot water than in cool water, hot water again cools faster than cold water.

Jim Haddocc's user avatar

  • $\begingroup$ Did you see the links I have mentioned? $\endgroup$ –  Yashbhatt Jul 18, 2014 at 16:53
  • $\begingroup$ 1)Notice that there is a point where the temperatures are equalized.Afterwards the rates should be the same 2)This perspective seems to apply in all liquids,but this not happens. $\endgroup$ –  elias2010 Apr 22, 2020 at 19:15
  • $\begingroup$ " This happens due to cooling affect of evapourisation. " Has no one ever thought of putting lids on the containers? $\endgroup$ –  Ray Butterworth Jul 20, 2022 at 13:22

It is because of convection effect. Convection is fluid (in our scenario) movement driven by temperature difference.

In this case grater temperature difference at the beginning - grater speed of fluid at the end. Fluid is cooling down near dish walls. This causes cooled water to sink faster in this part of dish.

Picture shows water movement in dish in cold environment. Arrows show fluid movement direction. Black arrow shows place where there is no heat exchange with environment. Blue arrows show where fluid is cooling down. Horizontal arrows are not important - fluid is not accelerated in this areas.

Water movement in dish in cold environment. Arrows show fluid movement direction. Black arrow shows place where there is no heat exchange with environment. Blue arrows show where fluid is cooling down. Horizontal arrows are not important - fluid is not accelerated in this areas.

A - container with water hotter at the beginning of the experiment

B - container with water colder at the beginning of the experiment

At the moment when water freezes speed of water circulation in A is grater than in B. Grater speed of fluid in dish - grater heat exchange with cold environment. Water circulation momentum causes inertia in (resistance in changing) heat exchange ratio.

Experiment:

Repeat experiment with dense 3D grid inside dishes. Such a grid that it prevents significant convection movements. Convection movement will still appear in each grid cell separately but effect will be negligible.

Evaporation effect is negligible.

It is worth to notice that no matter what the truth is there is such a moment where:

average heat of A = average heat of B

This is exact moment where A overtake B. In this state A has some other property in better condition then B.

In my opinion those properties are (as I mentioned before): momentum of fluid and temperature distribution (A has temperature more levelled then B, B has bigger temperature gradients).

Pawel Welsberg's user avatar

  • $\begingroup$ Can you please elaborate on this point? And what do you say about the other effects mentioned in other answers here. $\endgroup$ –  Yashbhatt Jul 22, 2014 at 4:47
  • $\begingroup$ @Pawel, I really like your proposal. It's definitely worth investigating. $\endgroup$ –  LMSingh Sep 27, 2014 at 20:12
  • $\begingroup$ 1)"those properties" can be expressed by entropy.But why the effect is not always occurs? 2)This perspective seems to apply in all liquids,but this not happens. $\endgroup$ –  elias2010 Apr 22, 2020 at 19:01

Sometimes it occurs: New Explanation .Initially-hot water has lost much of its ordered clustering (higher entropy) and, if the cooling time is sufficiently short, this will not be fully re-attained before freezing. Experiments on the low-density water around macromolecules have shown that such clustering processes may take some time.Entropy reduction curves function of temperature S=f(T) appear retardation (lagging) relative to entropy growth curves.At any temperature point T the entropy S=mclnT during cooling is more than this during heating.The water after was heated and recooled at the initial temperature,has more entropy than before it was heated.This means that molecules have now the same average kinetic energy,but thermal motion before heating was more oriented by the structure mentioned above.Recooling random collisions are more possible leading to faster temperature’s reduction New explanation .

elias2010's user avatar

  • 1 $\begingroup$ Dear elias2010. For your information, Physics.SE has a policy that it is OK to cite oneself, but it should be stated clearly and explicitly in the answer itself, not in attached links. $\endgroup$ –  Qmechanic ♦ Apr 22, 2020 at 23:32

Consider this arxiv. Most of people intuit that since at certain point the T(t) curves (temperature vs time) of systems with different initial temperatures cross - the cooled system somehow has memory or is aware of its cooling protocol. This last statement is what's causing trouble. During the cooling process the definition of temperature is ill defined since the system is not in equilibrium. The cooling protocols merely transfer one equilibrium state to another (asymptotically, of course), through non-equilibrium states. Since the initial conditions are considered to be substantially different - the path of the two systems through the non equilibrium states differ substantially as well. Different paths in general traversed at different rates. This is the solution to the paradox, and invitation to investigate optimal cooling protocols.

Alexander's user avatar

  • $\begingroup$ Interesting link. My intuition has always been that the masses and timescales involved in the Mpemba effect are large and slow enough that the usual definition of temperature should still be quite good. A peer- reviewed paper which treats the non-equilibrium details quantitatively will make a good read. $\endgroup$ –  rob ♦ Jan 9, 2019 at 12:01
  • $\begingroup$ 1)This perspective seems to apply in all liquids 2)What about the randomness and low repeatability of the effect in water? $\endgroup$ –  elias2010 Apr 22, 2020 at 18:55

If you are putting a metal ice tray onto a freezer shelf which is covered with a significant layer of frost, then a hot tray will melt the frost (which is a fair insulator) putting the tray into direct contact with the freezer element.

R.W. Bird's user avatar

The surface tension, and therefore the droplet size in free-fall, of water is dependent on its temperature. Therefore, water thrown into the air will form smaller droplets with greater total surface area at higher water temperatures. A larger ratio of surface area to mass allows for more rapid heat transfer with the air through which it falls by conduction.

In very cold air, it is possible to throw a cup of boiling water into the air and have it turn into snow before it lands, due to the very high total surface area of the droplets thus formed. Under these conditions, it is feasible for a cup of cold water, which would form larger droplets, to take longer to reach freezing point.

Unfortunately it is not presently cold enough here to conduct such an experiment.

Chromatix's user avatar

Your Answer

Sign up or log in, post as a guest.

Required, but never shown

By clicking “Post Your Answer”, you agree to our terms of service and acknowledge you have read our privacy policy .

Not the answer you're looking for? Browse other questions tagged thermodynamics water temperature molecules atomic-physics or ask your own question .

Hot network questions.

  • I'm looking for a series where there was a civilization in the Mediterranean basin, which got destroyed by the Atlantic breaking in
  • Under physicalism, should I still be sad if my murdered wife is replaced with a perfect clone?
  • Build the first 6 letters of an Italian codice fiscale (tax identification number)
  • What is the U.N. list of shame and how does it affect Israel which was recently added?
  • How might a physicist define 'mind' using concepts of physics?
  • Looping counter extended
  • Can I paraphrase an conference paper I wrote in my dissertation?
  • Has ever a country by its own volition refused to join United Nations, or those which havent joined it's because they aren't recognized as such by UN?
  • Dispute cancellation fees by 3rd-party airline booking
  • How to make sub "array" to aligned with the outer "array" in equation in LaTeX?
  • Inductance after core saturation
  • What is the frequentist's Bayesian prior for a coin with unknown bias
  • Where do UBUNTU_CODENAME and / or VERSION_CODENAME come from?
  • Transformer with same size symbol meaning
  • Which ability checks are rolled for a shove attack?
  • Python matrix class
  • How can I hang heavy bikes under a thick wooden shelf?
  • Are there any jobs that are forbidden by law to convicted felons?
  • What does "however" mean in this sentence? Does it mean ''still'' or "moreover"?
  • Simple explanation of what "preferential lattice expansion due to excessive magnesium adsorption", means
  • How do you keep the horror spooky when your players are a bunch of goofballs?
  • Handling cases of "potential" ChatGPT-generated reviews in non-anonymous program committees (as a PC member)
  • Is obeying the parallelogram law of vector addition sufficient to make a physical quantity qualify as a vector?
  • Asymptotic behavior of a ratio involving Bessel K functions

does hot water or cold water freeze faster experiment

does hot water or cold water freeze faster experiment

Advertisement

From Liquid to Solid: How Long Does It Take Water to Freeze?

  • Share Content on Facebook
  • Share Content on LinkedIn
  • Share Content on Flipboard
  • Share Content on Reddit
  • Share Content via Email

instant ice

You may have seen videos of what looks like an ordinary bottle of cold water hanging out and minding its own watery business until bam! Someone taps it against the table and the whole bottle turns instantly to ice . What is this dark wizardry?

It isn't magic but instead science that causes the bottled water to completely freeze — and some pretty simple science at that. So, how long does it take water to freeze ? Let's find out.

The Mystery of Nucleation

Freezing point fun facts, quick freezing faqs and myth busting.

When any substance changes state — like liquid water changing to solid ice — the process involves nucleation . It’s the anchor that creates the first ice crystal and then promotes the rapid formation of more.

Heterogeneous Nucleation

This happens when there are impurities, like dust, present in the water, providing the necessary nucleus for ice formation in water exposed to freezing temperatures. Ice crystals then form throughout the liquid, turning our water into solid ice over time.

Homogeneous Nucleation

Pure water has no impurities, so without a nucleus to kickstart the freezing process, the water becomes supercooled. This allows the water to freeze faster when exposed to an external nucleus, making the magic of "instant" ice possible.

Water famously becomes completely frozen at 32 degrees Fahrenheit (0 degrees Celsius). But when water is devoid of impurities, like in purified bottled water, the freezing process requires even colder temperatures.

So, if you place bottles of purified water in the cold air of a freezer and leave them a couple of hours, they'll still be liquid because pure water with no nuclei in it freezes at minus 43.6 degrees Fahrenheit (minus 42 degrees Celsius). It's now a supercooled liquid, which does indeed sound super cool.

Let's Make Some Instant Ice!

Ready to freeze water? Grab some water bottles and place them in your freezer. Make sure it's undisturbed for a few hours, getting it to that supercooled state. The exact freezing time? Typically, it takes about two-and-a-half to three hours .

Once the wait is over, remove the bottles with care. Then shake one or whack it on the table.

Anything can act as a nucleus at this point — air bubbles, a slight dent in the bottle. Any little change will be enough to cause homogenous nucleation. Once that disturbance is present, the uniform water molecules will freeze completely and so quickly that it looks instant.

An alternative to the whacking or shaking method is to pour the supercooled water over an ice cube. The cube will serve as the nucleus, and you'll be able to create a little tower of ice as you pour.

Which Freezes Faster, Hot or Cold Water?

An interesting phenomenon known as the Mpemba effect suggests that under certain conditions, hot water freezes faster than cold water. Crazy, right?

Do Different Ice Trays Affect the Freezing Process?

Absolutely! A metal ice cube tray, for instance, might speed up the process of freezing water for solid ice cubes compared to a plastic ice tray, because metal ice cube trays conduct heat (and the lack of it). Oversized ice cube trays, on the other hand, might take longer simply due to the larger volume.

What's the Ideal Water for Instant Ice?

Bottled or purified water is typically best for this icy experiment, but tap water, depending on how treated it is, can sometimes work too.

This article was updated in conjunction with AI technology, then fact-checked and edited by a HowStuffWorks editor.

Please copy/paste the following text to properly cite this HowStuffWorks.com article:

  • Login | Register

Have 10% off on us on your first purchase - Use code NOW10

Free shipping for orders over $100

Available for dispatch within 2 days

Free gift with purchase of over $100

Check out with Paypal and Afterpay

What Freezes first… Hot or Cold Water?

Follow FizzicsEd 150 Science Experiments:

You Will Need:

  • One cup of cold water, 100mL in volume
  • One cup of hot water, 100mL in volume
  • One Stopwatch
  • One Stirrer
  • A pen and paper

What freezes first, hot or cold water Science Experiment - setupmaterials

  • Instruction

What freezes first, hot or cold water Science Experiment - end results

Stir both water cups the same amount of time. Place both cups of water inside your freezer and start the timer.

does hot water or cold water freeze faster experiment

Keep checking at 5 minute intervals to see which freezes first.

Student using a pen to write on paper on a black table cloth

Record your observations. What happened?

LN2 sprinkler by Holly SciFest Africa Grahamstown March 2015

School science visits since 2004!

– Curriculum-linked & award-winning incursions.

– Over 40 primary & high school programs to choose from.

– Designed by experienced educators.

– Over 2 million students reached.

– Face to face incursions & online programs available.

– Early learning centre visits too!

Orange read more button

Get the Unit of Work on Heat Energy here!

  • What actually is heat?
  • How does heat move through different materials?
  • How does heat change the properties of materials and more!

Includes cross-curricular teaching ideas, student quizzes, a sample marking rubric, scope & sequences & more

Teacher showing how to do an experiment outside to a group of kids.

Online courses for teachers & parents

– Help students learn how science really works

Why Does This Happen:

Hmmmm, you’ve completed an experiment whose results are quite tricky to explain. Did you find that the hot water froze first? Under some conditions this can happen…

We started with two containers of water, which were identical in shape and held identical amounts of water. The only difference between the two was that the water in one was at a higher (uniform) temperature than the water in the other. Of course, if the hot water had started at 99.9° C, and the cold water at 0.01° C, then clearly under those circumstances, the initially cooler water would have frozen first. However, under some conditions the initially warmer water will freeze first — if that happens, you have seen the Mpemba effect which describes the phenomenon.

How does it work you might ask? Several ideas have been put forward and no-one really is sure as to which effect plays the biggest role:

1. As the initially warmer water cools to the freezer temperature, it may lose significant amounts of water to evaporation. The reduced mass will make it easier for the warmer water to cool and freeze than the colder water.

2. A convection current may have been setup in the warmer water. As the warmer water cooled it lost heat primarily through the surface of the liquid faster than the colder water. This is due to a great temperature difference between the cold freezer air and the warm water. The water from the bottom of the cup then rose to the water surface, bringing more heat energy to the cold freezer air. As the current is greater in the warmer water than the cold water, a greater amount of liquid got exposed to the cold freezer air. Think of a fan forced oven, circulating the hot air through the oven heats the oven faster than just allowing the air to sit still… bakers have known this over a thousand of years!

More on temperature and water rising or falling 3. The surrounding air around the cups may have more movement around the warmer cup, therefore drawing heat energy away from the warmer cup more effectively.

4. Warm water holds less dissolved gas than cold water. There have been some suggestions that the presence of dissolved gases impede the production of convection currents in the colder water.

5. The cold water may have supercooled , therefore not forming a solid as quick as the hot water.

Quote: “I often put boiling water in the freezer. Then whenever I need boiling water, I simply defrost it.” -Gracie Allen.

Variables to test.

More on variables here

  • What happens if you add salt to the water?
  • Does it make a difference if you change the volume of water?

A man with a glove above a liquid nitrogen vapour cloud

Learn more!

From colour changes to slimy science, we’ve got your kitchen chemistry covered! Get in touch with FizzicsEd to find out how we can work with your class.

does hot water or cold water freeze faster experiment

Chemistry Show

Years 3 to 6

Maximum 60 students

Science Show (NSW & VIC)

Online Class Available

White tree in a green pot

Magic Crystal Tree Science Kit

Stem full day accelerator - primary.

Designed from real classroom experiences, this modular day helps you create consistently effective science learning that directly address the new curriculum with easily accessible and cost-effective materials.

Be Amazing! How to teach science, the way primary kids love.

Be Amazing! How to teach science, the way primary kids love.

Love science subscribe.

Receive more lesson plans and fun science ideas.

SCIENCE PARTIES

Calendar of events.

does hot water or cold water freeze faster experiment

HIGH SCHOOL Science@Home 4-Week Membership 12PM: March 2024

does hot water or cold water freeze faster experiment

Price: $50 - $900

does hot water or cold water freeze faster experiment

PRIMARY Science@Home 4-Week Membership 2PM: March 2024

does hot water or cold water freeze faster experiment

Light and Colour Online Workshop, Jan 18 PM

Light and colour online workshop, jan 18 am.

does hot water or cold water freeze faster experiment

Lego Robotics, Sydney Olympic Park Jan 2024

does hot water or cold water freeze faster experiment

Creative Coding, Sydney Olympic Park Jan 2024

does hot water or cold water freeze faster experiment

Creative Coding, Sydney Olympic Park July 11 2023

Price: $100

does hot water or cold water freeze faster experiment

Fizzics Education STEAM Day: Robots vs Dinosaurs, Lalor, Apr 14

Price: $45 - $50

Creative Coding, Sydney Olympic Park April 14 2023

Science@home after school 4-week membership: march 2023.

Price: $40 - $1200

28 thoughts on “ What Freezes first… Hot or Cold Water? ”

Very helpful and useful for kids and everyone and it’s easy to understand.

Thanks Amal, we’re glad that you’ve been using these science activities!

can you please answer these 2 questions: What’s the independent, dependent & controlled variable of this experiment? And what equipments have you used for this experiment?

The independent variable for this experiment was the temperature of the water (this was the variable that we could manipulate). The dependent variable was the time it took for the water to change from liquid to solid (as this depends on the initial temperature of the water). The controlled variables were the freezer temperature, the cup size, the volume of the water used in each test and the amount of stirring in each cup. More details on variable testing here

You can find the details on the materials needed at the top of this page. Have fun with this experiment!

what are some risks that can take place when doing this experiment

If the water is close to boiling you really need to be careful. Also, water spills are a slipping hazard.

Is this an actual good experiment that will work in class

Hi Ben, give it go and see what happens! All the best.

is it ok to do in school

Sure thing, just check with your teacher first and let them handle the hot water. Have fun!

i don’t have it yet,but it sounds like its super fun! : )

Its an interesting experiment! Please let us know what your results were… do they conform to the theory above? Have fun!

How hot should the hot water be?

Hi! Try cups with different amounts of heat – you might find that there is a particular temperature where the result can be opposite 🙂 We like to use near-boiling vs room temperature, but you could try every temperature as a series of tests (as long as you’re safe with water over 55 degrees celsius). Have fun!

Hi, can you please describe the science involved in this investigation

Hi Jae! This is based on the Mpemba effect. Check out further details here

The cold water froze first for me, why?

This has been a tough one for us to reproduce too! We’ve found that there can be slight differences within the freezer itself, whereby placement of the cups in relation to the vents being a crucial factor. Did you try it again?

wow, this experiment was really surprising at first but now that I have read the science behind it, it makes a lot more sense.

Great to hear that you were able to confirm this experiment! It’s a strange one to observe 🙂

Just did the experiment with my son. I was fully expecting the hot water to freeze first, but it ended up being the cold water that froze. Cold started to freeze after 20 minutes. At 40 minutes cold is solid and hot is probably a third of the way frozen. Disappointed that the results didn’t turn out as described in the experiment. Maybe there needs to be more controls included in the experiment. I made sure to use the same cup and I put the cups in the same spot of the freezer. It would be good to have a suggested temperature for both.

Hi Alisa, I’m glad to hear you and your son have tried this experiment! Freezing water may seem commonplace, so you’d think that scientists would have this all figured out…but in fact, the Mpemba Effect is still being researched and redefined! From fancy labs where things are monitored and controlled down to the molecular level to high school classrooms, scientists see different results. This is because the effect is only sometimes observed under “some conditions”, as we have stated in the experiment description. The latest research suggests that convection currents, nucleation sites, supercooling and dissolved gases/ions all might play a part, and have complex interactions. This means that even though you have used the same cup and put it in the same spot in the freezer (great variable control, by the way!), our freezers at home may still introduce inconsistencies as it cycles on and off to maintain temperature (usually -10 to -18 degrees Celcius), or have the cooling vent in a location which favours the cold or hot sample (top or bottom may influence the outcome). How we pour the water into the cup and walk it over to the freezer and place it might make a difference. One scientist even wrote that “two samples of water taken from the same bottle may differ significantly”! While these are factors that are very hard to control at home, this is such an interesting experiment to attempt because this is not an experiment where it will “work” or “fail”. At Fizzics we don’t always get the same results either, but we love figuring out why we see each observation! The authors of this article had some success with around 50 degrees Celcius difference in sample temperatures. Fizzics scientists have observed the effect in the past with nearly boiling (80-100 degrees) vs. cold tap water.

I loved this experiment

Glad that you enjoyed it!

Hi this is perfect for my students because I looked it up on Google and it was perfect and like to say thank you guys for putting this up because I need to show it to my students and I love how it’s very good experience

Hi this is perfect for my students

Fantastic! Please let us know how you go 🙂

Leave a Reply Cancel reply

Your email address will not be published. Required fields are marked *

School Comments View All

Fizzics Education curated a thoughtful and hands-on experience for the children, incorporating practical, skill-based learning activities and followed by a science presentation at the end of the event involving liquid nitrogen. This was delivered safely and effectively, capturing both the children and the parents for the duration of the presentation.

Fizzics Education ran a show today at our school and it was wonderful. He was a great facilitator and the show was age appropriate and well done.

I just wanted to pass on how much the staff and students really enjoyed it and how perfect it was to launch our science week activities. The students were enthralled, educated and entertained – a perfect trifecta!

Thanks so much for presenting at our school on Monday. Our students enjoyed the show.

Fizzics Education Awards

blue writting saying Australian Small Business Champion with a white ackground and and image at the top

  • Free Resources

Free Chemistry Book! Sign-up to our newsletter and receive a free book!

Female Accountant Apply Here

Physics teacher apply here, science teacher apply here, view all vacancies, join our team apply here.

Send us an Email at [email protected]

does hot water or cold water freeze faster experiment

Phone Number: 1300 856 828

Email: [email protected], address: unit 10/55 fourth ave blacktown, nsw 2148, australia.

  • Privacy & Legal Policy
  • Copyright Notice
  • Terms of Trade
  • Cookie Policy

Copyright 2024 Fizzics Education . All rights reserved.

This website uses cookies to improve user experience. By using our website you consent to all cookies in accordance with our Cookie Policy .

Get more science with our newsletter!

Thank you for looking to subscribing to our newsletter 🙂 Through this service you’ll be first to know about the newest free experiments, science news and special offers.

PLUS: Get a free Kitchen Chemistry Booklet with >20 experiments, how to use variables plus a handy template!

Click the image to preview

Please select an ebook!

does hot water or cold water freeze faster experiment

Kids Edition

does hot water or cold water freeze faster experiment

Parent Edition

does hot water or cold water freeze faster experiment

Teacher Edition

Please fill out the details below and an email will be sent to you. Once you get that just click on the link to confirm your subscription and you're all done!

First Name *

Last Name *

Email Address *

Phone Number

Subscribe as a Teacher?

Preschool Teacher

Primary Teacher

High School Teacher

Vacation Care or Library

Subscribe as a Parent?

Enquiry Form

Extra things, products that might interest you.

Rainbow Fireworks Glasses

Rainbow Fireworks Glasses

A child holding a Helicopter spiral top with a smile

Helicopter Spiral Top

Fly Back Gliders

Fly Back Glider

  • Grade Level

Where are you located?

  • New South Wales
  • Australian Capital Territory

Location not listed?

Which grade level are you teaching.

  • Whole School
  • Teacher Professional Development
  • Special School Events
  • Early Childhood
  • Kindergarten

Which broad syllabus outcome you want to teach?

Green circle with the letters BS written in the middle

What is the age range of the attendee?

  • Age 5 and up
  • Age 6 and up
  • Age 7 and up
  • Age 8 and up
  • Age 9 and up
  • Age 10 and up
  • Age 11 and up
  • Age 12 and up

General Enquiry Form

Check if you require a live online class.

Subscribe for special offers & receive free resources?

How did you hear about us?

Choose a program *

Choose from school show *

* Please select a value!

* Please add a value!

Date required *

Time required *

IMAGES

  1. Does hot water freeze faster than cold water?

    does hot water or cold water freeze faster experiment

  2. Hot VS. Cold

    does hot water or cold water freeze faster experiment

  3. Why hot water freezes faster than cold water?

    does hot water or cold water freeze faster experiment

  4. 52 Simple Science Experiments For Kids

    does hot water or cold water freeze faster experiment

  5. Does Hot Water Freeze Faster Than Cold Water?

    does hot water or cold water freeze faster experiment

  6. This Is Why Hot Water Freezes Faster Than Cold Water • AwesomeJelly.com

    does hot water or cold water freeze faster experiment

VIDEO

  1. why does hot water freeze faster than cold

  2. Did you know, why hot water can freeze faster than cold water #intrestingfacts #facts #intresting

  3. Does hot water freeze faster than cold water? #familyfunpack #twintime #twins #comedy #viral

  4. Hot Water Can Freeze Faster Than Cold Water

  5. Mysterious Fact

  6. Is it quicker to freeze your ice cube tray using hot or cold water? #shortvideo #viral #water

COMMENTS

  1. The Physics of Why Hot Water Sometimes Freezes Faster Than Cold Water

    Under certain forces from the laser, the hottest beads cooled faster than the lower temperature beads. The research suggests that the longer path from a higher temperature to the freezing point ...

  2. Does hot water freeze faster than cold water?

    It's an age-old question with a simple answer: no. By Sarah Jensen. Since the time of Aristotle, researchers and amateur scientists alike have batted about the counterintuitive theory that hot water freezes faster than cold. The notion even has a name: the Mpemba effect, named for a Tanzanian schoolboy who in 1963 noticed that the ice cream ...

  3. Is It True that Hot Water Freezes Faster than Cold Water or that Cold

    "Cold water does not boil faster than hot water. The rate of heating of a liquid depends on the magnitude of the temperature difference between the liquid and its surroundings (the flame on the ...

  4. Does Hot Water Really Freeze Faster Than Cold Water?

    As the video above explains, the phenomenon of hot water freezing faster than cold water is known as the Mpemba effect, named after Erasto Mpemba, a Tanzanian student who in 1963 was making ice cream as part of a school project. The students were meant to boil a mixture of cream and sugar, let it cool down, and then put it in the freezer.

  5. A new experiment hints at how hot water can freeze faster than cold

    Sometimes hot water can freeze faster than cold. A new experiment based on tiny glass beads may help explain why. In physics, chilling out isn't as simple as it seems. A hot object can cool more ...

  6. The Mpemba Effect: Does Hot Water Really Freeze Faster Than Cold Water

    The Mpemba effect is a physics concept that postulates that when hot water and cold water are placed in the identical freezing environment, the hot water will freeze faster than the cold water. Erasto Mpemba noted that when his class was making ice cream, he placed a near-boiling blend of sugar and milk (which is mostly water) into a freezer ...

  7. Why Does Hot Water Freeze Faster Than Cold Water?

    The most commonly proposed hypothesis - and one that's probably somewhat responsible for the effect - is that hot water evaporates more quickly, losing mass and therefore needing to lose less heat in order to freeze. However, scientists have also demonstrated the Mpemba effect with closed containers where evaporation doesn't take place.

  8. Does Hot Water Freeze Faster Than Cold Water?

    First, all water evaporates, which means that the liquid (water) "disappears" and becomes a vapor, or gas. Hot water evaporates at a much faster rate than cold water. This means that the bowl with hot water actually had less water than the bowl with cold water, which helped it freeze more quickly. Second, convection (the transfer of heat within ...

  9. You really can freeze hot water faster than cold*

    Hot water really can freeze faster than cold water, a new study finds. Sometimes. Under extremely specific conditions. With carefully chosen samples of water. New experiments provide support for a ...

  10. Quanta Magazine

    Or perhaps external factors come into play: A layer of frost in a freezer can act as an insulator, keeping heat from leaking out of a cold cup, whereas a hot cup will melt the frost and cool faster. Those explanations all assume that the effect is real — that hot water really does freeze faster than cold. But not everyone is convinced.

  11. Can hot water freeze faster than cold water?

    Yes—a general explanation. Hot water can in fact freeze faster than cold water for a wide range of experimental conditions. This phenomenon is extremely counterintuitive, and surprising even to most scientists, but it is in fact real. It has been seen and studied in numerous experiments . Although this phenomenon has been known for centuries ...

  12. Mpemba effect

    Temperature vs time plots, showing the Mpemba Effect. The Mpemba effect is the name given to the observation that a liquid (typically water) which is initially hot can freeze faster than the same liquid which begins cold, under otherwise similar conditions. There is disagreement about its theoretical basis and the parameters required to produce ...

  13. Investigating the 'Mpemba Effect': Can Hot Water Freeze Faster than

    Introduction. It may seem counterintuitive, but folk wisdom and a body of published evidence agree that, under some conditions, warmer water can freeze faster than colder water (for an excellent review on the subject, see Jeng, 2005). This phenomenon has been known for a long time, but was rediscovered by a Tanzanian high school student, Erasto ...

  14. Why Hot Water Freezes Faster Than Cold—Physicists Solve the ...

    The Mpemba effect is the observation that warm water freezes more quickly than cold water. The effect has been measured on many occasions with many explanations put forward.

  15. It's True: Hot Water Really Can Freeze Faster Than Cold Water

    Hot water really can freeze faster than cold water, a new study finds. Sometimes. Under extremely specific conditions. With carefully chosen samples of water. New experiments provide support for a ...

  16. Does Hot Water Freeze Faster Than Cold Water?

    Determining whether or not hot water can freeze faster than cold water may seem like a no-brainer. After all, water freezes at 0 degrees Celsius. And wouldn't water hot enough to kill E. coli ...

  17. Is It True Hot Water Freezes Faster Than Cold?

    There is no definitive explanation for why hot water may freeze faster than cold water. Different mechanisms come into play, depending on the conditions. The main factors appear to be: Evaporation : More hot water will evaporate than cold water, thus reducing the amount of water remaining to be frozen. Mass measurements lead us to believe this ...

  18. Here's how hot water might freeze faster than cold

    It seems logical. But some experiments have suggested that under the right conditions, hot water can freeze faster than cold. Now chemists offer a new explanation for how this might happen. What they don't do, however, is confirm that it actually does occur. The speedier freezing of hot water is known as the Mpemba effect.

  19. physics

    The hot water will evaporate much faster when it is very shallow and spread out. It has less total mass to retain heat and a lot more surface area to cool it and evaporate it. The hot water will evaporate reducing mass and will then freeze faster than the cold water. The relative humidity in the air will also be a variable to consider ...

  20. thermodynamics

    Now this is where thwe cooling effect of evapourisation takes place. As the water evapourates, it takes away some heat thus cooling the hot water . so as a equation we can write it as: Rate of cooling of hot water = cooling effect of evapourisation + cooling effect of freezer Rate of cooling of cold water = cooling effect of freezer.

  21. Does Hot Water Freeze Faster Than Cold Water?

    Does hot water freeze faster than cold water? Turns out, the answer to this question is a lot trickier than it seems!Hosted by: Michael Aranda-----Doobl...

  22. From Liquid to Solid: How Long Does It Take Water to Freeze?

    Which Freezes Faster, Hot or Cold Water? An interesting phenomenon known as the Mpemba effect suggests that under certain conditions, hot water freezes faster than cold water. Crazy, right? ... Bottled or purified water is typically best for this icy experiment, but tap water, depending on how treated it is, can sometimes work too. ...

  23. What Freezes first… Hot or Cold Water? : Fizzics Education

    The only difference between the two was that the water in one was at a higher (uniform) temperature than the water in the other. Of course, if the hot water had started at 99.9° C, and the cold water at 0.01° C, then clearly under those circumstances, the initially cooler water would have frozen first. However, under some conditions the ...